JUN 2018

You might also like

Download as pdf or txt
Download as pdf or txt
You are on page 1of 110

Get more from: MplusX Qbank (https://member.mplusx.

com/)
Facebook Page: (https://www.facebook.com/mplusxqbank/)

Q 1A study is conducted to check the efficacy of antibiotics prior to suturing wound what is increase in
absolute risk reduction?
Values were 2. 4
96. 98
Options were
1%
2%
33%
50%
98%
26 yr old pregnant female at 34 weeks AOG, comes to you bcoz her son had a case of chicken pox,prior to
getting pregnant she did VZV IG Negative and requests for vaccination.Asking most appropriate next step
1 Repeat VZV IG antibody test
2 Give VZV vaccination and IG
3 No need to vaccinate now
4 Vaccinate after delivery
5 i dont remember not relevant

A 16 year old female presents with Sickle cell anaemia and heavy menstrual bleeding. What is the best
treatment of choice?
A. Combined Oral Contraceptive Pill (COCP)
B. Inj Depoprovera
C. Mirena
D. IUCD
E. Implanon

Q7 a patient presents with 3 months history of nausea and epigastric discomfort he has icteric sclera he
consumes 50-60 grams of Alcohol. Lab investigations were given all liver enzymes including total and
conjugated bilirubin were raised and serum lipase was also raised on Usg there was dilatation of
intrahepatic channels not extrahepatic no other abnormality what you will do?
ERCP
CT abdomen
MRCP
Ena

Seems to b ca head of pancreas

given x ray of pneumonia in 10 in years old child asking about next investigation:
a CT chest
b blood culture
c pleural aspirate
d manteaux testdoscopic ultrasound .

pt for ect and refuses while agitated she refuse to eat for days ur action.
a consent from her son
b consent from mental health tribunal
c ect regardless her wish as it will save her d. do nothing

What is correct about cpr?


A. 2 artificial breathing every 5 compression

AfraTafreeh.com
Get more from: MplusX Qbank (https://member.mplusx.com/)
Facebook Page: (https://www.facebook.com/mplusxqbank/)

B. Check pluse every 2 mins


C. 100 compression in 1 min

Patient with controlled DM 2 from 5 years. What will you check in routine every 3 months. ?
GFR
Thyglyceride
Microalbuminuria

In amedex ppl saying hb aic

Microalbumin urea annually

3 cm mass in left lobe thyroid without any invation to lymph node. It says papillary ca in stem.
Total thyroidetomy
Lobectomy
Radio iodine ablation
Ablation with thyroidectomy

Tx total thyroidectomy followed by iodine ablation

Baby was born normal, suddenly become limp, cyanosed,heart rate 30 per minute, amniotic fluid was
slightly meconium stained, what is the most important next step?
A)intubation
B)nasopharangyeal aspiration of meconium
C)oxygen deliver by bag and mask

58 year old lady worried for postmenopausal fracture due to osteoporosis,,,she was treated for ca breast
4 years back,,,,u did scan and T value -2.8 treatment….
CAL/VIT D
ALENDRONATE
HRT
RaLOXIFEN.

Old woman with urgency and nocturia for a long time. Now having incontinence for like a month. She
couldn't reach the toilet downstairs. She also has bilateral knee OA. What is the best management for
her?
A. Fluid restriction at night
B. Install a camode in her bedroom.
C. Oxybutynin
D. Paracetamol

23 year old woman, choatic anxious agitated, multiple wrist lacerations. This happens post breakup with
boyfriend. Admitted. Shouts on nurses. Found with inflated bp apparatus around neck. Cause:
A. Passive agression
B. Acting out
C. Schizophrenia
D. Genuine attempt of suicide

Diabetes ulcer on the foot pic.


diabetic pt, non healing ulcer, pic 2 ulcers in medial malleolus, and foot pulses not pulpable. X ray already
done. what to do for Mx?

AfraTafreeh.com
Get more from: MplusX Qbank (https://member.mplusx.com/)
Facebook Page: (https://www.facebook.com/mplusxqbank/)

A) blood culture
B) white cell scan
C) swab from ulcer
D) venous dupplex USG
E) MRI foot

15 years Boy previously a good student now present with agitation , violent , admit to occasional
marijuana use, multiple needle marks on forearm but don’t want to talk about it.

a) Respiridone
b) Paroxetin
c) Methylphenidite

Its adolescent depression accorging to symptoms

. Patient had history of pancreatitis. Some time ago .now presented with distension tenderness visible
vessels percussion positive 37.5 temp liver enzymes raised, confused and slightly drowsy. Dx
1. Bud chiari
2. Pancreatitis
3. Alcoholic cirhosis ( dont remember other options)
4. Spontaneous bacterial peritonitis

You are a doctor in a town, where 6 people out of 100 are non-smoker. What are the
chances of stroke. The chances of stroke in smoker is 50%more than non-smoker. Now
the pharmaceutical company is introducing a medicine which reduces the chances of
stroke by 1/3rd in smoker population. What is the percentage of the smoker population
will get stroke due to drug?
A. 4%
B. 6%
C. 9%
D. 12%
E. 20%

HRT risk per year; which is correct?


a) Breast CA 10% increase per year (2.3% per year)
b) Gall stones 2% decrease per year (increase)
Heart disease 1% increase per year

Noreen new questions

AfraTafreeh.com
Get more from: MplusX Qbank (https://member.mplusx.com/)
Facebook Page: (https://www.facebook.com/mplusxqbank/)

20th june

AfraTafreeh.com
Get more from: MplusX Qbank (https://member.mplusx.com/)
Facebook Page: (https://www.facebook.com/mplusxqbank/)

Q 1A study is conducted to check the efficacy of antibiotics prior to suturing


wound what is increase in absolute risk reduction?
Values were 2. 4
96. 98
Options were
1%
2%
33%
50%
98%
Q2 patient wants to be your friend on social media what you should do?
Tell them they can discuss only medical related issues
They can message you directly
Accept the request but don’t contact directly
Decline the request
Delete your social media site
Q3 resident while doing laparoscopy causes injury to liver surgeon comes
and control but it results in laparotomy what should be done?
Tell the patient he made error
Tell the patient it was complementary complication
Don’t tell as patient is recovered
Q4 surgeon does some harm to patient while doing surgery and ask
resident to tell the patient nothing happens what he should do
Should report about surgeon(only option I m able to recall)
Q5 nurse tells dr she is hep b positive what should be done?
Terminate her job
Ask her to tell patients about her status before any procedure
No need to change her role
2 more options
Q6 A locum at private gp service finds patient record incomplete and some
orders about patient treatment are not followed what you should do?
Maintain records and patient call up system
Ask consultants when they return
Urge patients to go to some other doctor
2 more options
Q7 a patient presents with 3 months history of nausea and epigastric
discomfort he has icteric sclera he consumes 50-60 grams of Alcohol. Lab
investigations were given all liver enzymes including total and conjugated
bilirubin were raised and serum lipase was also raised on Usg there was

AfraTafreeh.com
Get more from: MplusX Qbank (https://member.mplusx.com/)
Facebook Page: (https://www.facebook.com/mplusxqbank/)

dilatation of intrahepatic channels not extrahepatic no other abnormality


what you will do?
ERCP
CT abdomen
MRCP
Endoscopic ultrasound
Q8 Chronic COPD patient want to quit smoking he has tried 4 times
previously but feels restless and agitated what is the strongest indication
for start of nicotine therapy?
His withdrawal symptoms
His duration of disease
Family history of smoking
Obesity
Q9 in a community there is increase violence because of alcohol use what
you should do as a medical practitioner
Financial and psychosocial support
Appoint police at high risk areas
Distribute pumflets to people on medical effects of excess alcohol
2 more options
Q10 there was a CT scan picture in which right kidney was normal but left
was abnormal distorted large mass most appropriate next was asked
Percutaneous biopsy
Repeat CT scan after 6 monts
MRI
Renal artery angiography
Left nephrectomy
Q11 mother is worried about excess wt gain in baby there is 100 gm wt
increase in one wk mother is restless and tired of baby care what should
you do?
Increase breast feed
Increase formula feed
Asses her about post natal depression
Refer him to lactational consultant
A lady comes with hypertension 135/85 waist circumference 92 cm And total cholesterol 5.5 what
will be the highest risk of developing cardiovascular event in next 5 years
Hypercholestrolemia
Hypertension
Obesity

AfraTafreeh.com
Get more from: MplusX Qbank (https://member.mplusx.com/)
Facebook Page: (https://www.facebook.com/mplusxqbank/)

Another q A lady comes with obesity what will be long term consequence of her obesity
Asthma
Diverticulitis
Ca coloon
Sleep apnoea

Anum file 21 ,6,2018


Hey everyone, I gave my exam today. I had lots of ECG and cts n pics. Do keep me in your prayers. All the
best.
1.New born after 7 day with shedding of umbilical cord, yellowish liquid and there is red fleshy papule,
with erythramatous base ... management?
a) apply Topical silver nitrate
b) referral
c) Excision
d) dressing and bandage
E) antibiotics
From <https://www.facebook.com/groups/1442556592721801/search/?query=umbilical%20granuloma>
2.Growth chart given with weight at birth 2.5kg, 3mo- 3kg and at 6mo -6kg. Mother says he is not feeding
well and gaining weight. After plotting the child's weight in the chart What to say to mother? (note-the
growth chart had no marking about percentile)
A)Childs weight is suboptimal and needs assessment
b)Childs weight is normal and Reassure
C)child's growth is more for his age.
If baby weigt doubles at 6 months its norsl
From <https://www.facebook.com/groups/1442556592721801/search/?query=growth%20chart>
3.39 weeks pregnant lady came with labour pain..she was put in left lateral position having oxygen mask in
place.iv fluids with syntocinon is running. Ctg was done which shows heart rate of 140 which dropped to 70 and
came back to 140 in 2min.asking next appropriate treatment.
k) A. Fetal scalp sampling
l) B. Continuuous ctg monitoring
m) C.stop syntocinon
n) D. C section
From <https://www.facebook.com/groups/1442556592721801/search/?query=3min%20stop%20synto>
4.Baby was born normal, suddenly become limp, cyanosed,heart rate 30 per minute, amniotic fluid
was slightly meconium stained, what is the most important next step?
A)intubation
B)nasopharangyeal aspiration of meconium
C)oxygen deliver by bag and mask
From <https://www.facebook.com/groups/1442556592721801/search/?query=bag%20and%20mask>
5.
70 year old lady with ca pancreas CT given.Features of Obstructive jaundice given with Labs and CT,
Ejection fraction <20% What will be the appropriate management
A. Laproscopic cholecystectomy
B. Whipples
C. Liver Biopsy
D. ERCP
From <https://www.facebook.com/groups/1442556592721801/search/?query=ct%20with%20obstructive%20jaundice>
6.
10 there was a CT scan picture (looked like supra renal mass on left kidney?!) in which right kidney was
normal but left was abnormal distorted large mass most appropriate next was asked
Percutaneous biopsy
Repeat CT scan after 6 monts
MRI
Renal artery angiography

AfraTafreeh.com
Get more from: MplusX Qbank (https://member.mplusx.com/)
Facebook Page: (https://www.facebook.com/mplusxqbank/)

Left nephrectomy

Left nephrectomy
From <https://www.facebook.com/groups/1442556592721801/permalink/1984090618568393/?comment_id=
1984160495228072&notif_id=1529549406668847&notif_t=group_comment_mention>
7.
In a community there is increase violence because of alcohol use what you should do as a medical
practitioner
Financial and psychosocial support
Appoint police at high risk areas
Distribute pumflets to people on medical effects of excess alcohol
Start alcohol anonymous group
From <https://www.facebook.com/groups/1442556592721801/permalink/1984090618568393/?comment_id=
1984160495228072&notif_id=1529549406668847&notif_t=group_comment_mention>
8.
Chronic COPD patient want to quit smoking he has tried 4 times previously but feels restless and agitated
what is the strongest indication for start of nicotine therapy?
His withdrawal symptoms
His duration of disease
Family history of smoking
Obesity
From <https://www.facebook.com/groups/1442556592721801/permalink/1984090618568393/?comment_id=
1984160495228072&notif_id=1529549406668847&notif_t=group_comment_mention>
9.a patient presents with 3 months history of nausea and epigastric discomfort he has icteric sclera he
consumes 50-60 grams of Alcohol. Lab investigations were given all liver enzymes including total and
conjugated bilirubin were raised and serum lipase was also raised on Usg there was dilatation of
intrahepatic channels not extrahepatic no other abnormality what you will do?
ERCP
CT abdomen
MRCP
Endoscopic ultrasound
From <https://www.facebook.com/groups/1442556592721801/permalink/1984090618568393/?comment_id=
1984160495228072&notif_id=1529549406668847&notif_t=group_comment_mention>
10.A study is conducted to check the efficacy of antibiotics prior to suturing wound what is increase in
absolute risk reduction?
Values were 2. 4
96. 98
Options were
1%
2%
33%
50%
98%
From <https://www.facebook.com/groups/1442556592721801/permalink/1984090618568393/?comment_id=
1984160495228072&notif_id=1529549406668847&notif_t=group_comment_mention>
11.
a given x ray of pneumonia in 10 in years old child asking about next investigation:
a- CT chest
b- blood culture
c- pleural aspirate
d- manteaux test
From <https://www.facebook.com/groups/1442556592721801/search/?query=child%20chest%20x%20ray%20what%
20investigation%20to%20do>
12.25 yr old female complain of Raynaud's phenomenon in cold,also having edema of bl feet,,bibasal
crepitation on lung bases ,heart on CXR silhute appearance .ana+, ENA neg..cz

AfraTafreeh.com
Get more from: MplusX Qbank (https://member.mplusx.com/)
Facebook Page: (https://www.facebook.com/mplusxqbank/)

2018 Page 2

AfraTafreeh.com
Get more from: MplusX Qbank (https://member.mplusx.com/)
Facebook Page: (https://www.facebook.com/mplusxqbank/)

crepitation on lung bases ,heart on CXR silhute appearance .ana+, ENA neg..cz
a.primary Raynaud's
b.cold agglitonin antibody
c.sle
d.limited sclerosis
From <https://www.facebook.com/groups/1442556592721801/search/?query=SLE>
13.SLE scenario. Pic of hand with rash. Most important investigation to reach the Dx?
a) anti Sm Abs
b) Anti Ro
c) Anti La
d)ANA
From <https://www.facebook.com/groups/1442556592721801/search/?query=SLE>
14.management of SLE long term :
a) immunosuppressive plus anti coags
b) immunosuppressive plus anti platelets
c) immunosuppressive plus antibiotics
d) anti coags plus anti platelets
From <https://www.facebook.com/groups/1442556592721801/search/?query=SLE>
15.Colles fracture recall but with different options xray at 6 week followup was normal and now pt having
mild pain and tingling sensations what to do
A. Xray again
B. splint for few days more
C. occupational therapy to check on him
D.exercise analgesics
E. MRI to see ligament
From <https://www.facebook.com/groups/1442556592721801/search/?query=wrist%20fracture>
16.A lady present to you and she tells that God tell her `Fast 40 days and time so that World War end
‘.She is orientated and decline her to test other function. What is your first line of management?
A) Zuclopenthixol
B) Diazepam
C) Quatiapine
D) RisperidoneConsta
E) Clozapine
From <https://www.facebook.com/groups/1442556592721801/search/?query=zuclopenthixol>
17.History of appendicetomy & cholecystectomy presented with 3 wk history of abd pain, distension. X-
ray shows dilated ascending colon, transverse colon & descending colon, no air. Rectum is empty.
A- Sigmoid volvulous
B- Adhesive IO
C- CA Sigmoid
From <https://www.facebook.com/groups/1442556592721801/search/?query=cecal%20volvulous>
18.one population , mean systolic BP is 115 and SD is 15 so what will be range of 95% population’s BP?
a) 100 to 130
b) 85 to 145
C)105- 140
From <https://www.facebook.com/groups/1442556592721801/search/?query=SD>
19.What is the best method of study for studying the relation between fatigue and accident in truck
driver?
a. RCT
b. Cohort
c. case-control study
d. Case report
e. systemic review
From <https://www.facebook.com/groups/1442556592721801/search/?query=fatigue%20accident>

AfraTafreeh.com
Get more from: MplusX Qbank (https://member.mplusx.com/)
Facebook Page: (https://www.facebook.com/mplusxqbank/)

2018 Page 3

AfraTafreeh.com
Get more from: MplusX Qbank (https://member.mplusx.com/)
Facebook Page: (https://www.facebook.com/mplusxqbank/)

From <https://www.facebook.com/groups/1442556592721801/search/?query=fatigue%20accident>
20.pt for ect and refuses while agitated she refuse to eat for days ur action.
a- consent from her son
b- consent from mental health tribunal
c- ect regardless her wish as it wil save her
D do nothing
From <https://www.facebook.com/groups/1442556592721801/search/?query=refuse%20to%20will>
21.post op agitation on 3rd day in a pt who drink 4-6 cans of beer everyday. smokes 20 packs of
cigarettes everyday. Pt tries to remove his Iv line cause?
a)Alcohol withdrwal
b)drug reaction
c)psychosis
d)dementia
From <https://www.facebook.com/groups/1442556592721801/search/?query=post%20agitation>
22.an old lady no h/o alcoholism.went through surgery for incarcerated femoral hernia.after 4 hour she
became agitated.spo2 86%.she wants climb the bed.after giving o2 via mask what do u next approach-
a.antibiotic
b.iv fluid
c.droperidol
d.anticoagulant
From <https://www.facebook.com/groups/1442556592721801/search/?query=post%20agitation>
23.Post op patient, agitated, trying to get out of bed, removing all IV infusion, all vitals normal except
SaO2 is 86% in room air. After giving oxygen by face mask, what is next:
Droperidol
Blood glucose level
CT
Intubate
IV antibiotics
From <https://www.facebook.com/groups/1442556592721801/search/?query=post%20agitation>
24. CT scan of diverticulitis in elderly woman with scenario of fever, LIF pain, mild leukocytosis, history of
constipation and peri colic abscess found. Most appropriate next treatment?(CT given)
A. Broad spectrum antibiotics with gut rest
B. Prednisolone
C. CT guided drainage
D. Laparoscopic surgery
E. sigmoid resection

2018 Page 4

AfraTafreeh.com
Get more from: MplusX Qbank (https://member.mplusx.com/)
Facebook Page: (https://www.facebook.com/mplusxqbank/)

E. sigmoid resection
From <https://www.facebook.com/groups/1442556592721801/search/?query=CT%20diverticulitis>
25.a man 45 yr has no family h/0 of cancer.came for PSA screening.wat advice youff give him?
a.psa is not specific for diagnosing carcinoma
b.no need for psa
c.psa from 50 yr
d.more than 50% chances to give false positive
From <https://www.facebook.com/groups/1442556592721801/search/?query=prostate%20cancer%20screening>
26.45 yrs old man with no family history of prostate cancer comes for screening advice and counselling .
He insist he wants a DRE.
A digital rectal exam showed a normal sized prostate with normal non nodular sulci. What is the next
important step
A. Reassurance
B. PSA after 2 weeks
C .TRUS
D. Abdominal usg
From <https://www.facebook.com/groups/1442556592721801/search/?query=prostate%20cancer%20screening>
27.ECG of SVT patient presented with dizziness and his BP 90/40 what’s your management ??
Dc cardiovertion
Iv adenosin
Iv amiodarone
Defibrillator
From <https://www.facebook.com/groups/1442556592721801/search/?query=ECG%20of%20SVT>
28.ECG of VT with WPW, Rx?
A IV adenosine
B digoxin
C IV sotolol
D carotid sinus massage
E Amiodarone
Right Answer : C IV sotolol

this is a clear case of WPW alternating with VT .. WPW is treatedwith Procaineamide , Amiodarone Or
Sotalol

WPW is not treated as SVT .. or u kill ur patient .. Only if it presents with Narrow-complex AV re-entrant
tachycardia then u treat as PSVT with Vagal maneuvers and Adenosine.

In WPW

If Unstable ==> Cardiovesrion


IF Stable?==> Procainamide or Sotalol then Amiodarone

From <https://www.facebook.com/groups/1442556592721801/search/?query=ECG%20of%20VT>
29.ECG of Pulseless elecctrical activity asking for mx.
30.Ecg of V.fib. Defib done but still no change. What next?
A. Continue CPr for 2 mi
B. Repeat defib

2018 Page 5

AfraTafreeh.com
Get more from: MplusX Qbank (https://member.mplusx.com/)
Facebook Page: (https://www.facebook.com/mplusxqbank/)

B. Repeat defib
C. Drug options
d.iv adrenalin
From <https://www.facebook.com/groups/1442556592721801/search/?query=ECG%20of%20Vfib>
31.Scenario with HF, raised JBP, fullness on lung percussion. On digoxin, thiazides all lots of other
medication. ECG was there with HF. Asked what to do next. Digoxin level was 1.5 (normal was 2.5)
A. Increase digoxin dose
B. Stop all medications and review after 3days
C. Increase thiazide
From <https://www.facebook.com/groups/1442556592721801/search/?query=digoxin%20dose>
32.PAIN IN THE BUTTOCKS&POSTERIOR THIGH&DEADLY LEG PAIN AFTER WALKING 100 M
ON NORMAL FLOOR &20 M ON UNEQUAL LAND WELL FELT PEDAL PULSATION NEXT?
A..DOPPLER LEGS
B.CT ANGIO.
C.MRI LUMBOSACRAL
D.X.RAY
Neurogenic pain
From <https://www.facebook.com/groups/1442556592721801/search/?query=neurogenic%20claudication>
33.patient with history of claudication, smoke 30 pack cigarette per day, drink alcohol, obese with
diabetic history. he refuse surgery although he was fit for it. He asked you for the appropriate advice that
will improve his symptoms of claudication?
A-Reduce smoking
B-reduce alcohol drinking
C-supervised exercise
D-control his hypercholesterolemia
From <https://www.facebook.com/groups/1442556592721801/search/?query=neurogenic%20claudication>
34.ECG – ST elevation in V1, V2, V3, no arrhythmia detected
Patient has history of chest pain 2 hour ago , symptom relieved and now pain 4/10. He is on atorvastatin
and aspirin. On admission, given oxygen. Most appropriate next step in management?
A. Nitrate
B. Metoprolol
C. Morphine
D. Clopidrogrel
E. Thrombolysis
From <https://www.facebook.com/groups/1442556592721801/search/?query=atorvastatin>
35.Aman taking so many medicine e.g metformin,aspirin, atorvastatin and metoprolol suddenly become
hypoglycemic ?what is the cause?
metoprolol
statin
aspirin
metformin
metoprolol masks symptoms of hypoglycemia
From <https://www.facebook.com/groups/1442556592721801/search/?query=atorvastatin>
36.A pregnant with hep C comes to you for advice ,what will reduce the risk of transmission of
hep C to her baby
C sec
Breastfeeding
Avoid Fetal scalp electrodes
From <https://www.facebook.com/groups/1442556592721801/search/?query=hep%20c>
37.You are a doctor in a remote area where a study is conducted to determine the prevalence of Hep B
the study included 200 patients and showed the following results 2008/ 2009

2018 Page 6

AfraTafreeh.com
Get more from: MplusX Qbank (https://member.mplusx.com/)
Facebook Page: (https://www.facebook.com/mplusxqbank/)

the study included 200 patients and showed the following results 2008/ 2009
Patients with Hep B Ag +ve only 10 / 12
Patients with +ve Hep B core antibody and Hepb Ag 50/ 7
Patients with Hep B core antibody +ve only 20/ 60
Calculate the incidence in
2010 in 1000 patients from this population (sure about 2010 given numbers)
75
50
300
375

June 23,2018

Raheel zafar

1.Woman patient has been booking late afternoon bookings for .. months. She feels lonely and later
become attracted to the doctor. One week later , she called the hospital for home visit for tonsillitis. What
will be ur advice?

a) Visit her home with surgery staff member


b) Explain her that she will not be seen by that doctor anymore
c) Tell her to come to emergency depertment
d) Ask her to call other medical services

. 4 years old girl. Mother notice purulent amount of blood in her underwear for 10 days. On examination ,
there is small amount of blood stained in vulva. What is ur first step in the management? Not developed
any pubic hair and no breast development.
a) Report child protective service immediately for sexual abuse.
b) Microsopic blood culture of the Swab from the vulva
c) oral Amoxil (not sure)

Old woman with urgency and nocturia for a long time. Now having incontinence for like a month. She
couldn't reach the toilet downstairs. She also has bilateral knee OA. What is the best management for
her?
A. Fluid restriction at night
B. Install a camode in her bedroom.
C. Oxybutynin
D. Paracetamol

52 years old woman with incontinence of urine during coughing and laughing. Her urodynamic study
shows that her incontinence occurs during low volume study. On examination she had mild rectocele. Her
symptoms showed some improvement after pelvic floor exercise but she could not satisfied this result.
What will your appropriate step management?
A-Bladder neck suspension
B-Topical Estrogen cream
C-Weighted vaginal cones
Do not remember

Men 35 years with history of difficulty in micturition for 2 years. Difficult to start and dribbling at the end
of micturition.
a) BPH
b) Bladder stone
c) Urethral stricture

AfraTafreeh.com
Get more from: MplusX Qbank (https://member.mplusx.com/)
Facebook Page: (https://www.facebook.com/mplusxqbank/)

d) Urethral valve
e) Cancer (not sure

Old man with urinary incontinence. Not severe but it causes a problem when he goes out. First step of Mx?
a) Pelvic floor exercise
b) Bladder train
c) Prazosin
d) Anticholinergic agen

Woman with yellow eye with facial palsy pic

Diabetes ulcer on the foot pic.


diabetic pt, non healing ulcer, pic 2 ulcers in medial malleolus, and foot pulses not pulpable. X ray already
done. what to do for Mx?

A) blood culture
B) white cell scan
C) swab from ulcer
D)venous dupplex USG
E) MRI foot

Rule out osteomyelitis

14 month baby who has history of roll over on 4 month, sitting without support from 8 month , claps
hand, plays peak a boo, now has started walking without support, moves things from one hand to other ,
babbling but no words...
normal development
Gross motor delay
Fine motor delay
Social delay
Speech delay

23 year old woman, choatic anxious agitated, multiple wrist lacerations. This happens post breakup with
boyfriend. Admitted. Shouts on nurses. Found with inflated bp apparatus around neck. Cause:
A. Passive agression
B. Acting out
C. Schizophrenia
D. Genuine attempt of suicide

15 years Boy previously a good student now present with agitation , violent , admit to occasional
marijuana use, multiple needle marks on forearm but don’t want to talk about it.

a) Respiridone
b) Paroxetin
c) Methylphenidite

. Patient had history of pancreatitis. Some time ago .now presented with distension tenderness visible
vessels percussion positive 37.5 temp liver enzymes raised, confused and slightly drowsy. Dx
1. Bud chiari
2. Pancreatitis
3. Alcoholic cirhosis ( dont remember other options)
4. Spontaneous bacterial peritonitis

AfraTafreeh.com
Get more from: MplusX Qbank (https://member.mplusx.com/)
Facebook Page: (https://www.facebook.com/mplusxqbank/)

. patient comes for dryness of eyes for 3 years.... all tests for sjogren positive. ANA(+) ,. Ask for the next tx
A Hydroxychloroquine
B steroid
C methotrexate
D hypomallose tears
E. prednisolone eye drops

12 week pregnant with lower abdominal pain. Normal nuchal usd at 11wk. Heavy PV bleeding, BP 80/50
incomplete miscarriage
Rupture ectopic
Missed miscarriage.

You are a doctor in a town, where 6 people out of 100 are non-smoker. What are the
chances of stroke. The chances of stroke in smoker is 50%more than non-smoker. Now
the pharmaceutical company is introducing a medicine which reduces the chances of
stroke by 1/3rd in smoker population. What is the percentage of the smoker population
will get stroke due to drug?
A. 4%
B. 6%
C. 9%
D. 12%
E. 20%

. 37 ek pregnant lady with mild pre-eclampsia for induction of labour.Bishop score 2.What will you do?
Elective LUCS
Induction with PGE 1 and ARM after 4 hours
Induction with PGE 2 and ARM after 12 hours.
Urgent LUCS
Syntocinon infusion

Mother 41+ 3 days with no abnormality. Previous child born by LSCS due to obstruction of labor. Bishop
score 2.
a) Induction with prostaglandin
b) Induction with ARM
c) LSCS
d) Admission for fetel examination
e) CTG after 1 week

72 year female come for routine check up. Her mother develop colon cancer at the age of 65 years. She
doesn’t have any bowel symptoms and haven’t done any screening test before. She want to know her
risk?
a) Nothing to do
b) Colposcopy
(sry.. forget other options

18. 72 year female come for routine check up. Her mother develop colon cancer at the age of 65 years. She doesn’t
have any bowel symptoms and haven’t done any screening test before. She want to know her risk? a) Colposcopy 2
yrly b) 5 yrly cervical cancer screening c) 5 yearly colon cancer screening d) mammogram year e) no screening
needed

AfraTafreeh.com
Get more from: MplusX Qbank (https://member.mplusx.com/)
Facebook Page: (https://www.facebook.com/mplusxqbank/)

A 44 years old man attends your OPD with complaint of increasing swelling over last 12 hours on his right
thigh. He is a diabetic and hypertensive and has been previously operated 6 weeks back for a fracture
femur. He is on Ramipril, Simvastatin, Warfarin, Amiodarone, NSAIDs, and antacids. On examination you
find that the right thigh is 4 cm greater in circumference than the left thigh and is tender. His temperature
is 37.8. Which of the following is the reason for his presentation?
A. Arterial embolus
B. Cellulitis
C. Deep vein thrombosis
D. Edema
E. Drug interaction

Woman 55 years came to doctor for regular medical check up. Her mother has CA cervix at 63 years.
Maternal aunt has breast CA at --- year . and father has colon cancer at 72 years. What to check her?

a) Colposcopy 2 yrly
b) 5 yrly cervical cancer screening
c) 5 yearly colon cancer screening
d) mammogram yearly

HRT risk per year; which is correct?


a) Breast CA 10% increase per year (2.3% per year)
b) Gall stones 2% decrease per year (increase)
c) Heart disease 1% increase per year

57yr male with waldenstorm macroglobulinemia (pic given)and myelodysplastic syndrome developed
pneumonia. Penicillin given. After 2 days he developed maculopapular rash with some pustular lesion
.What would lead to definitive diagnosis:
A. Skin biopsy
B. Immunoflourescense of lesion
C. Blood culture
D. Marrow biopsy
E. swab of lesion

1. Patient post laparotomy on PCA (patient controlled analgesia) giving morphine 1g now found
that he is drowsy but responds to command wht next
Cease drug
Change Pca dose to 0.5 mg
change to nurse administered on demand
Oxygen

Pt posted for surgery on femur on multiple meds. Which should be stopped at least a week
before

a. Ramipril

b. Metformin

c. Ibuprofen

d. Thiazides

e. Amitryptilline

AfraTafreeh.com
Get more from: MplusX Qbank (https://member.mplusx.com/)
Facebook Page: (https://www.facebook.com/mplusxqbank/)

Woman with BMI 32. Smoke 10 cigarettes per day?. Has premenstrual headaches etc . Apart from
reducing weight , what is her Mx?
a) Smoking habit
b) History of premenstrual headache
c) ???

what is seen in Bulimia nervosa ?


a) ankle oedema
b) tachycardia
c) vitiligoyu 6

Man whose hand deeply injured by screw driver , 5 weeks ago, he found supercial laceration and give
diphtheria tetanus toxoid , now , the hand is swollen and painful. What to do beside antibiotics?
a) Elevation
b) Aspiration
c) Tetanus toxoid
d) Tetanus immunoglobulin only
e) Dpt

A man complaint of ‘feeling something in his body‘ and anxiety. BP160/- . Give beta blockers
a. Investigate for pheochromocytoma
b. Investigate for hyperthyroidism
c. Arrange psychiatric counselling

past history of dvt pt on UFH perioopertively n switch to lmw heparin after 5 days undergone rt hip
surgery develop dvt after 10 days that what to do
In investigation only platelets r decreased
A- ffps
B- vit k
C- cease heparin and switch to other anticoagulant
D- platelets infusion

Woman with CCF , taking digoxin and perindopril regularly. Long list of lab result given. ECG show
pulmonary congestion(given) and she has progressive dyspnea

a) Add furusemide
b) Add
c) Cease digoxin and give K+ supplement
d) Cease perindopril and give K+ supplement
e) Urgent rhymthm inversion?

Boy in MVA , seatbelt injuty to left chest. On examination , dimished sound on left sided , condition not
improving . on arrival , chest and abdomen pain with BP 90/60 , HR – 110/min. Give IV fluid in hospital.
After one hour , he complaint of shoulder pain with BP 80/60 , HR . 120/min .no pneumothorax seen in
xray. Mx ?

a) Chest drain
b) Intubation and IPPV
c) Needle thoracocentesis
d) Laparotomy
e) Laproscopy

AfraTafreeh.com
Get more from: MplusX Qbank (https://member.mplusx.com/)
Facebook Page: (https://www.facebook.com/mplusxqbank/)

Woman present with back pain after lifting heavy weight.. Ask about degree , she said 4/10. Pain not
radiating to other site. What factors can cause chronic pain?

a) Being woman
b) No radicular pain
c) Pain 4/10
d) ??

Woman fall on outstretched hand at the back door of her house. Her xray on 6 weeks postfracture heal
finely. She came to followup on 8 week post fracture only show mild intermittent pain. Mx?

a) Repeat xray wrist


b) Sling for better stability?
c) CT
d) Arrange occupational therapy for back door safety

a young girl had 1 week history of depressed mood , 2 weeks back she had sexual assault.
A.acute stress reaction
B.PTSD
C. malingering disorder

, The symptoms that define ASD overlap with those for PTSD. One difference, though, is that a PTSD
diagnosis cannot be given until symptoms have lasted for one month. So A h

X ray pic with man persistant dry cough, dyspnea . heard PSM murmur.x ray pic. Dx?
a) Pulmonary hypertention
b) Pulmonary TB
c) Rheumatic fever

overdosed on 25mg indapimide (diuretic dose)? Or something. Was admitted and not able to sleep
despite benzodiazepam. Nurse found her staring at ceiling and other times agitated. Cause:
a) dementia
b) depression
c) catatonia

it should b delirium

1. scenario of a lady who become agitated and confused at shopping center for 1-2 mins. She has h/o
same attacks . and during attack she stares blankly, doesn’t respond to any command followed by
conusion. Sometime during conversation with relatives it happens and she doesn’t follow the
conversation. What is ur dx?
a) Panic attack
b) GAD
C) Complex partial seizures
d)PTSD

a child with red & swollen scrotum not tender no fever


A.idiopathic scrotal edema
B.hydrocele
C. varicocele

AfraTafreeh.com
Get more from: MplusX Qbank (https://member.mplusx.com/)
Facebook Page: (https://www.facebook.com/mplusxqbank/)

pt with carcinoma of ceacum asking most likely symptom??


A.altered bowel habit
B.lethargy/ tiredness
C.black coloured stool
D.recurrent colicky abdominal pain

1. your in a rural area and 4 cases of trachoma come to you then you find out extra 20 how can you treat
acute indexes ? Apart from hand washing , what will u give?
b-oral azithromycin
c-doxycyclin
d-don’t do anything
e-penicillin

. Old man with recurrent falls in nursing home, he is found to have many bruises in head, ECG showed
sinus tachy, multiple ventricular ectopics and ventricular hypertrophy (written), on enam/thiazide
combination, BP standing 150/90 standing and systolic 90 in sitting, what invx for diagnosis?
D) holter monitoring
E) 24 hr BP
F) Repeated BP measurements with postural change
G) CT head

40 years old man complain from persistent abnormal thoughts that’s making him washing his hands at
least 10 times after touching the key or door.he is on SSRI but want to try non pharmacological therapy to
help him get rid of this thoughts
Teach him how to avoid touching the doors or key
Explain him that this thoughts is normal to relieve his anxiety
Refer him to insight oriented psychotherapy
Have psychological evaluation

Dx?

AfraTafreeh.com
Get more from: MplusX Qbank (https://member.mplusx.com/)
Facebook Page: (https://www.facebook.com/mplusxqbank/)

Thrombosed hemorriods

Doctor after night shift from emergency department went to bar. He found his colleagues there and some
are having illicit drugs. They have to go to hospital for duty 2 hours later. What will u do?
a) Report to director of emergency department
b) Tell directly to those colleagues
c) Leave and said nothing
d) Inform APHRA

Child comes with excessive vomiting for several times with abdominal pain.No features of diarrhea.
Finger prick test for glucose is normal .What is the immediate danger to him?
A. Cerebral oedema
B. Hypoglycemia
C. Hypokalaemia
D. Hyperkalaemia

24yr old male with dysphagia and history of asthma in childhood. Endoscopy showed narrowing and
inflammation of middle esophagus, biopsy showed eosinophils in mucosa. What is the best treatment?
a. Oral Antibiotic
b. PPI
c. Budesonide

Old age pt presented with c/o epigastric burning nd regurgitation his investigations shows followin result
what will you do for most appropriate management? 1.oesophageal dilation
2 laproscopic cardiomyotomy
3 nifedipine
(exact pic)

AfraTafreeh.com
Get more from: MplusX Qbank (https://member.mplusx.com/)
Facebook Page: (https://www.facebook.com/mplusxqbank/)

Hereditary Hypercholesterolemia...which is found?


A)Corneal arcus
B)Tendon xanthoma
C)Periorbital xanthelesma
D)Fatty liver

Egyptian woman who doesn’t understand English have to do endoscopy procedure. Her daughter can
speak English fluently and husband understand the procedure well. Who should interpret?

a) Her daughter
b) Her husband
c) official interpreter
d) non medical staff from hospital
e) 2 outside general practitioners?

Patient with ECG of AF. But the vital signs are BP-80/60 and unstable. Which of the following is the best
management??
a- Digoxin
b- Verapamil
c- Cardioversion
d- Sotolol
e- Disepynamide

.Cystic fibrosis case, what will be the chance to have disease child if one of them is carrier?
a) 0%
b) 25%
c) 50%
d) 50% of all male & no female

AfraTafreeh.com
Get more from: MplusX Qbank (https://member.mplusx.com/)
Facebook Page: (https://www.facebook.com/mplusxqbank/)

cyst in anterior neck(pic given), no pain, lump for 6 wks had h/o Sx of oesophagus 10 years ago and
another surgery(can’t remember) 2 years ago. Dx
a. branchial cyst
b. epidermoid cyst
c. metastatic cancer

Epidermoid cysts, also called sebaceous, keratin, or epithelial cysts, are small, hard lumps that
develop under the skin. These cysts are common. They grow slowly. ... Epidermoid cystsare often
found on the face, head, neck, back, or genitals.Thesecysts often develop in response to skin
trauma, HPV infection, acne, or excessive exposure to the sun

Woman with 2-month h/o affected mood, in customer service role, missing work, can’t be bothered to go
to work, gets teary eyed often, recently hit her car to the pole, not paying attention, worries in the night
about losing her job, does not listen to her favorite music in the car, married 1 month ago to a caring
person.
a) Adjustment disorder
b) Dysthymia
c) Major depressive episode
d) Generalized anxiety disorder
e) Borderline Personality disorder

Patient with history of DVT, HTN with DM patient on warfarin, metformin, statin and then add
amiodarone, 2 days later suddenly got swelling at right thigh and become painful, rednessand Temp 37.8.
Right thigh size is 4 cm larger than left. Diagnosis?
A. Rhabdomylosis
B. Hematoma
C. DVT
D. Cellulitis
E. Drug reaction

AfraTafreeh.com
Get more from: MplusX Qbank (https://member.mplusx.com/)
Facebook Page: (https://www.facebook.com/mplusxqbank/)

Couple came for counseling of Schizophrenia. Husband’s mother has long term Schizo. He is around 25
yrs old and is completely normal till now. No symptoms in the wife. What are the chances of Schizo in the
child?
a. 10 fold increase in risk
b. More than 20 fold increase in risk
c. There is no such risk in the child
d. There is an increased chance of Schizo spectrum disorders in the child

30-year old lady came for regular antenatal visit at 36 or 38 weeks of gestation (don't remember
exactly). She had no problems and everything was normal except a foetal bradycardia of 60/min. An
emergency c section was done but the infant still had the bradycardia of 60/min. What is the cause?
A. anticardiolipin
B. factor V Leiden
C. Protein C
D. Lupus anticoagulant
E.anti Ro

A little less redder than this pic ^^ Tx

a) prednisolone
b) timolol
c) chloramphenicolManage

Dx secliritis

Malaria senario. . Asking which of the following most likely to exclude malaria??
A.Thrombocytopenia
B.daily fever
C.splenomegaly

AfraTafreeh.com
Get more from: MplusX Qbank (https://member.mplusx.com/)
Facebook Page: (https://www.facebook.com/mplusxqbank/)

58yr od male pt, since six month watery diarrhea mucus or blood ,with 3 kg weight loss.presented with a
hx of increasing difficulty to stand from sitting And walk upstairs. What is the cause?
a) Thyrotoxicosis
b) IBD
c) celiec
d) campylobacter jujeni
e) bowel cancer

A 7 years old girl with soiling in her underwear, she tells lies about accidents at school and hide her soiled
underpants and cry and quarrels with her brother when he calls her smelly. What is your diagnosis?
a)Regression
b)Conduct disorder
c)Oppositional Defiant Disorder
d)Delayed milestones
e)Depression

an infant presented with a discharge from the umbilicus. On cleaning a red granulomatous lesion was see,
treatment
a. topical silver nitrate
b. surgical excision
c. steroid ointement
d. antibiotic

diverticulitis scenario. Ct was given. What next


a. antibiotic and bowel rest
b. surgical exploration
c. colonoscopy
d. laparoscopy and procee

a middle aged female presents with lt wrist pain after colliding her car with a pole. On examination her lt
pupil was constricted and also had decreased field of vision which she was unaware of. What is the next
app in?
a. ct head
b. iop measurement
c. slit lamp examination

Old woman 75 years of age is admitted to the hospital following a community acquired pneumonia. She
received antibiotics and is well on the 5th day. She suddenly developed rigors, chills and high grade fever
on the 4th day. What is the most likely explanation?
a. Hospital acquired pneumonia
b. IV cannula related bacteremia
c. empyema
d. Pulmonary embolism

AfraTafreeh.com
Get more from: MplusX Qbank (https://member.mplusx.com/)
Facebook Page: (https://www.facebook.com/mplusxqbank/)

Heart failure case man with AF on Ramipril, metoprolol, digoxin. Went on trip for 2 weeks and stopped
his medication. Now with bilateral oedema up to knee in legs. And ankle swellings Lungs is clear. And
the patient is not dyspneic, What is the most appropriate management?
A. Serum creatinine and elecrtrolyte
B. Echo

Family with a newborn baby moved to a small village in rural Australia, father called up GP to inform wife
sad all the time, low mood, crying but taking good care of baby, cannot get her to the hospital as he is busy
with new work, what next:
A. Make a home visit
B. Send a social worker to check on woman
C. Insist husband to get the woman & baby to clinic
D. Admit the woman, baby can be with dad
E. Wait for the husband to call back again

A patient has surgery and on 10th post op day she developed dyspnea & agitated, he being a heavy
drinker and a smoker. What is the next appropriate thing to do?
A. ABGA
B. Chest Xray
C. Blood alcohol
D. CTPA
E. Ultrasound

A patient on Thyroxine. TSH is low. What investigation to do?


CT scan
TSH and T4
Glucose
Electrolyte and cr
Ct scan (pituitary dysfunction )

if increase tsh and low Thyroxine then thyroid issue

and if thyroxine low and tsh low then pituitary issue

Mother with 2 pre-school children said that ‘she doesn’t want to take care of her children anymore’ . what
to do?

a) Report to child protective service


b) Arrange mother for psychiatrist referral
c) ??

Man came to psychiatrist for …. His examination are all normal. He has history of criminal record for
assault , theft etc . After counselling section, he confided to the doctor that he wants to kill his ex-partner.
What next?
a) Warn his ex-partner about it

AfraTafreeh.com
Get more from: MplusX Qbank (https://member.mplusx.com/)
Facebook Page: (https://www.facebook.com/mplusxqbank/)

b) Send him for forensic assessment


c) 3-4 continuous councelling session with him

nform ex partner.if she stayed any authorised hostel then informed their secuirity n police as well

6 years old boy contact with a chicken pox friend in school, he is neither vaccinated nor infected
previously, first step of mx
a. check serology
b. immunoglobulin
c. give vaccine to the child

25 years old man came to ask for the advice about the PSA screening in Prostate Malignancy as his
grandfather diagnosed at the age of 70 and his neighbor now diagnosed to have as he is now 65.What
information will u give to him?
a. Screening is not indicated in this age
b. DRE is the good testing than PSA in this age
c. Can do now
d. It has high false positive rate

AfraTafreeh.com
Get more from: MplusX Qbank (https://member.mplusx.com/)
Facebook Page: (https://www.facebook.com/mplusxqbank/)

55 years man whose friend has prostate CA. ask for screening. His DRE is normal. What next?

a) Serum PSA
b) TRUS
c) Come back when symptoms appear

Man with surgery for clerk 1 metastatic melanoma , excise about 2mm margin of normal cell.
Malignant cell is around 1.5mm. He doesn’t have any other symptoms. Mx?

a) Regular observation
b) Axillary LN biopsy

80 yr old lady with features of carcinoma of caecum.What would be the most easily found symptoms??
A. Tiredness,lassitude,fatigue
B. RIF mass
C. Altered bowel habit
D. Bleeding PR
E. Melaena

Child,post viral ITP. Non blanchable petechial rashes. No other abn finding or bleeding. On FBE Hb n WBC
normal. PLATS 35×10 9. Treatment?
A. Strict bed rest
B. Steroids
C. Immunoglobulins

Eye completely closed with redness and fever, O/E difficulty in eye movements asking for investigation
A. CT head
B. Opthalmoscopic examination
C. Eye swab
D. Blood culture

Symptoms include eyelid pain, discoloration, and swelling; orbital cellulitis also causes fever, malaise, proptosis,
impaired ocular movement, and impaired vision. Diagnosis is based on history, examination, and CT or MRI.
Treatment is with antibiotics and sometimes surgical drainage
A ( orbital cellulitis) ( This is a surgical emergency. After consultation with the ENT surgeons and
ophthalmologists, an urgent CT scan should be arranged to differentiate those patients with an associated
abscess (usually subperiosteal) from those without. This should be discussed with the radiologist who will ask for
coronal views. Imaging should pay particular attention to the orbital and frontal regions as the abscess may be
small. Ref RCH)
Woman has regular menses for 4-6 wk interval, pain on left iliac fossa for 3 days, aggravated while
urination, her lmp was 9 days back, she had similar episode 4 wks back, which lasted for 3 days, cause?
a. Ovulation
b. complicated ovarian cyst
c. dysmenorrhea
d. early PID

Adrenal mass 5cm,incidental. Next?


A. Surgery
B. Observe

AfraTafreeh.com
Get more from: MplusX Qbank (https://member.mplusx.com/)
Facebook Page: (https://www.facebook.com/mplusxqbank/)

C. biopsy

The most common long term complication of streptococus pneumonia meningitis ?


a) cerebral palsy
b) deafness
c) epilepsy

An ambulance brought you a young lady who lost consciousness with body temperature
of 40° (hyperthermia) and BP was 155/90 when they found her collapsed at a friend’s birthday party.
What is next in Mx?
Naloxone
Benzodiapines
IV Normal Saline.
Cold blanket and Cool N saline

Old lady with mild dermentia , took medicines regularly. Now, she said she doesn’t want to take medicine
. His son also told that she seems well without taking her meds. What to do?

a) Accept patient’s wish


b) Send medical staff daily to give her meds
c) Warn her that she cant go home if she doesn’t take these meds

AfraTafreeh.com
Get more from: MplusX Qbank (https://member.mplusx.com/)
Facebook Page: (https://www.facebook.com/mplusxqbank/)

Patient who have performed IVF came for follow-up. How can know if it is successful? (sth like that)

a) Transvaginal USG
b) Perform APPA?
c) Measure B hcg

A pregnancy test is performed approximately two weeks after your egg retrieval. Pregnancy symptoms are not a
reliable sign of pregnancy success or failure, because symptoms may come and go. Bleeding is also more
common following IVF. If you discover vaginal bleeding after the transfer, it does not mean that the procedure
was unsuccessful. We will you ask you to get a blood pregnancy test (hCG level), approximately 12 days after
embryo transfer. You’ll take this test even if you’re bleeding,. When the test is positive, you will return for a follow-
up test two to three days later. The test is to confirm that the level of hCG is rising appropriately.
Young woman comes to you complaining of heavy menstrual bleeding with pain. She describes that clots
of blood are lost in the first few days of menses with severe back pain. You ask her for some investigations
that need to be performed. She gets irriated and says she just wants a treatment. Which of the following is
the apporpraite next treatment ?
a. Tranexemic acid from day 1 of menses
b. Mefanimic acid from day 1 of menses
c. Mirena
d. Oral contraceptive pills

Girl (age not mention) stay with her foster mother. She is normally shy and not talk much but lately she
has unusual sexual behavior with her neighbor (sth like that). Mother is upset and decide to send her
back to her original foster care. Management?
a) Examine the general development of the girl
b) Referral to child psychiatrist
c) Arrange to send her back to foster
d) Arrange the case conference with mother and child proctective service

June 20,18This is some of my questions. Thanks to Sayar U Tun Kyaw , my family


and friends.

Best of luck everyone! J

1.Woman patient has been booking late afternoon bookings for .. months. She feels lonely and
later become attracted to the doctor. One week later , she called the hospital for home visit for
tonsillitis. What will be ur advice?

a) Visit her home with surgery staff member

b) Explain her that she will not be seen by that doctor anymore

c) Tell her to come to emergency depertment

AfraTafreeh.com
Get more from: MplusX Qbank (https://member.mplusx.com/)
Facebook Page: (https://www.facebook.com/mplusxqbank/)

d) Ask her to call other medical services

1. 4 years old girl. Mother notice purulent amount of blood in her underwear for 10 days. On
examination , there is small amount of blood stained in vulva. What is ur first step in the
management? Not developed any pubic hair and no breast development.

a) Report child protective service immediately for sexual abuse.

b) Microsopic blood culture of the Swab from the vulva

c) oral Amoxil (not sure)

2. . Old woman with urgency and nocturia for a long time. Now having incontinence for like a
month. She couldn't reach the toilet downstairs. She also has bilateral knee OA. What is the best
management for her?
A. Fluid restriction at night
B. Install a camode in her bedroom.
C. Oxybutynin
D. Paracetamol

3. 52 years old woman with incontinence of urine during coughing and laughing. Her urodynamic
study shows that her incontinence occurs during low volume study. On examination she had mild
rectocele. Her symptoms showed some improvement after pelvic floor exercise but she could not
satisfied this result. What will your appropriate step management?

A-Bladder neck suspension

B-Topical Estrogen cream

C-Weighted vaginal cones

Do not remember

4. Men 35 years with history of difficulty in micturition for 2 years. Difficult to start and
dribbling at the end of micturition.

a) BPH

b) Bladder stone

c) Urethral stricture

AfraTafreeh.com
Get more from: MplusX Qbank (https://member.mplusx.com/)
Facebook Page: (https://www.facebook.com/mplusxqbank/)

d) Urethral valve

e) Cancer (not sure)

5. Old man with urinary incontinence. Not severe but it causes a problem when he goes out. First
step of Mx?

a) Pelvic floor exercise

b) Bladder train

c) Prazosin

d) Anticholinergic agent

6. Women with UTI and she suffer from significant persistent pain. She is also depressed,
insomnia , low mood, etc. Mx?

a) Tramadol

b) Diazepam

c) Citalopram

d) Diclofenac

Woman with yellow eye with facial palsy pic (same option)

PICTURE OF A LADY WITH DARK YELLOW EYES, FACIAL PALSY FEATURES, PAIN IN
THE EAR WITH DISCHARGE?
A. PREDNISOSLONE
B. FAMCICLOVIR
C. ANTIBIOTICS
D. CHLORAMPHENICOL

Prompt treatment of Ramsay Hunt syndrome can ease pain and decrease your risk
of long-term complications. Medications may include:

• Antiviral drugs. Medications such as acyclovir (Zovirax), famciclovir (Famvir)


or valacyclovir (Valtrex) often help combat the chickenpox virus.

AfraTafreeh.com
Get more from: MplusX Qbank (https://member.mplusx.com/)
Facebook Page: (https://www.facebook.com/mplusxqbank/)

• Corticosteroids. A short regimen of high-dose prednisone appears to boost


the effect of antiviral drugs in Ramsay Hunt syndrome.

• Anti-anxiety medications. Drugs such as diazepam (Valium) can help relieve


vertigo.

• Pain relievers. The pain associated with Ramsay Hunt syndrome can be
severe. Prescription pain medications may be needed

7. Diabetes ulcer on the foot pic.

ray diabetic pt, non healing ulcer, pic 2 ulcers in medial malleolus, and foot pulses not
pulpable. X already done. what to do for Mx?

A) blood culture

B) white cell scan

C) swab from ulcer

D) venous dupplex USG

E) MRI foot

8. 14 month baby who has history of roll over on 4 month, sitting without support from 8 month ,
claps hand, plays peak a boo, now has started walking without support, moves things from one
hand to other , babbling but no words...

normal development

Gross motor delay

Fine motor delay

Social delay

Speech delay

23 year old woman, choatic anxious agitated, multiple wrist lacerations. This happens post
breakup with boyfriend. Admitted. Shouts on nurses. Found with inflated bp apparatus around
neck. Cause:
A. Passive agression
B. Acting out

AfraTafreeh.com
Get more from: MplusX Qbank (https://member.mplusx.com/)
Facebook Page: (https://www.facebook.com/mplusxqbank/)

C. Schizophrenia
D. Genuine attempt of suicide

15 years Boy previously a good student now present with agitation , violent , admit to occasional
marijuana use, multiple needle marks on forearm but don’t want to talk about it.

a) Respiridone

b) Paroxetin

c) Methylphenidite

1. Patient had history of pancreatitis. Some time ago .now presented with distension tenderness
visible vessels percussion positive 37.5 temp liver enzymes raised, confused and slightly drowsy.
Dx
1. Bud chiari
2. Pancreatitis
3. Alcoholic cirhosis ( dont remember other options)
4. Spontaneous bacterial peritonitis

2. patient comes for dryness of eyes for 3 years.... all tests for sjogren positive. ANA(+) ,. Ask for
the next tx
A Hydroxychloroquine
B steroid
C methotrexate
D hypomallose tears

E. prednisolone eye drops

12 week pregnant with lower abdominal pain. Normal nuchal usd at 11wk. Heavy PV bleeding,
BP 80/50

incomplete miscarriage

Rupture ectopic

Missed miscarriage.

AfraTafreeh.com
Get more from: MplusX Qbank (https://member.mplusx.com/)
Facebook Page: (https://www.facebook.com/mplusxqbank/)

level You are a doctor in a town, where 6 people out of 100 are non-smoker. What are the

chances of stroke. The chances of stroke in smoker is 50%more than non-smoker. Now

the pharmaceutical company is introducing a medicine which reduces the chances of

stroke by 1/3rd in smoker population. What is the percentage of the smoker population

will get stroke due to drug?

A. 4%

B. 6%

C. 9%

D. 12%

E. 20%

3. 37 ek pregnant lady with mild pre-eclampsia for induction of labour. Bishop score 2.What will
you do?

Elective LUCS

Induction with PGE 1 and ARM after 4 hours

Induction with PGE 2 and ARM after 12 hours.

Urgent LUCS

Syntocinon infusion

Mother 41+ 3 days with no abnormality. Previous child born by LSCS due to obstruction of
labor. Bishop score 2.

a) Induction with prostaglandin

ccddddddddddddddddddd
d dcxxb) Induction with ARM

c) LSCS

d) Admission for fetel examination

e) CTG after 1 week

AfraTafreeh.com
Get more from: MplusX Qbank (https://member.mplusx.com/)
Facebook Page: (https://www.facebook.com/mplusxqbank/)

72 year female come for routine check up. Her mother develop colon cancer at the age of 65
years. She doesn’t have any bowel symptoms and haven’t done any screening test before. She
want to know her risk?

a) Nothing to do

b) Colposcopy

(sry.. forget other options)

1. A 44 years old man attends your OPD with complaint of increasing swelling over last 12 hours
on his right thigh. He is a diabetic and hypertensive and has been previously operated 6 weeks
back for a fracture femur. He is on Ramipril, Simvastatin, Warfarin, Amiodarone, NSAIDs, and
antacids. On examination you find that the right thigh is 4 cm greater in circumference than the
left thigh and is tender. His temperature is 37.8. Which of the following is the reason for his
presentation?
A. Arterial embolus
B. Cellulitis
C. Deep vein thrombosis
D. Edema
E. Drug interaction

Woman 55 years came to doctor for regular medical check up. Her mother has CA cervix at 63
years. Maternal aunt has breast CA at --- year . and father has colon cancer at 72 years. What to
check her?

a) Colposcopy 2 yrly

b) 5 yrly cervical cancer screening

c) 5 yearly colon cancer screening

d) mammogram yearly

HRT risk per year; which is correct?

a) Breast CA 10% increase per year(2.3% per year)

b) Gall stones 2% decrease per year (increase)

AfraTafreeh.com
Get more from: MplusX Qbank (https://member.mplusx.com/)
Facebook Page: (https://www.facebook.com/mplusxqbank/)

Heart disease 1% increase per year

57yr male with waldenstorm macroglobulinemia (pic given)and myelodysplastic syndrome


developed pneumonia. Penicillin given. After 2 days he developed maculopapular rash with some
pustular lesion .What would lead to definitive diagnosis:

A. Skin biopsy

B. Immunoflourescense of lesion

C. Blood culture

D. Marrow biopsy

E. swab of lesion

1. Patient post laparotomy on PCA (patient controlled analgesia) giving morphine 1g now found
that he is drowsy but responds to command wht next
Cease drug
Change Pca dose to 0.5 mg
change to nurse administered on demand
Oxygen

2. Pt posted for surgery on femur on multiple meds. Which should be stopped at least a week
before

a. Ramipril

b. Metformin

c. Ibuprofen

d. Thiazides

e. Amitryptilline

Woman BMI 26 . has .. surgery for some cancer and came for follow up. She is suffering from
sleep apnoea, osteoporosis , diverticular disease , .. cancer , asthma. Doctor advice her to reduce
weight. Which of the following is the complication of her obesity?

a) Sleep apnoea

b) Osteoporosis

AfraTafreeh.com
Get more from: MplusX Qbank (https://member.mplusx.com/)
Facebook Page: (https://www.facebook.com/mplusxqbank/)

c) Diverticular disease

d) Cancer

e) Asthma

Woman with BMI 32. Smoke 10 cigarettes per day?. Has premenstrual headaches etc . Apart
from reducing weight , what is her Mx?

a) Smoking habit

b) History of premenstrual headache

c) ???

what is seen in Bulimia nervosa ?

a) ankle oedema

b) tachycardia

c) vitiligo

Man whose hand deeply injured by screw driver , 5 weeks ago, he found supercial laceration and
give diphtheria tetanus toxoid , now , the hand is swollen and painful. What to do beside
antibiotics?

a) Elevation

b) Aspiration

c) Tetanus toxoid

d) Tetanus immunoglobulin only

e) Dpt

A man complaint of ‘feeling something in his body‘ and anxiety. BP160/- . Give beta blockers

a. Investigate for pheochromocytoma

b. Investigate for hyperthyroidism

AfraTafreeh.com
Get more from: MplusX Qbank (https://member.mplusx.com/)
Facebook Page: (https://www.facebook.com/mplusxqbank/)

c. Arrange psychiatric counselling

past history of dvt pt on UFH perioopertively n switch to lmw heparin after 5 days undergone rt
hip surgery develop dvt after 10 days that what to do

In investigation only platelets r decreased

A- ffps

B- vit k

C- cease heparin and switch to other anticoagulant

D- platelets infusion

Woman with CCF , taking digoxin and perindopril regularly. Long list of lab result given. ECG
show pulmonary congestion(given) and she has progressive dyspnea

a) Add furusemide

b) Add

c) Cease digoxin and give K+ supplement

d) Cease perindopril and give K+ supplement

e) Urgent rhymthm inversion?

Boy in MVA , seatbelt injuty to left chest. On examination , dimished sound on left sided ,
condition not improving . on arrival , chest and abdomen pain with BP 90/60 , HR – 110/min.
Give IV fluid in hospital. After one hour , he complaint of shoulder pain with BP 80/60 , HR .
120/min .no pneumothorax seen in xray. Mx ?

a) Chest drain

AfraTafreeh.com
Get more from: MplusX Qbank (https://member.mplusx.com/)
Facebook Page: (https://www.facebook.com/mplusxqbank/)

b) Intubation and IPPV

c) Needle thoracocentesis

d) Laparotomy

e) Laproscopy

Woman present with back pain after lifting heavy weight.. Ask about degree , she said 4/10. Pain
not radiating to other site. What factors can cause chronic pain?

a) Being woman

b) No radicular pain

c) Pain 4/10

d) ??

Woman fall on outstretched hand at the back door of her house. Her xray on 6 weeks postfracture
heal finely. She came to followup on 8 week post fracture only show mild intermittent pain. Mx?

a) Repeat xray wrist

b) Sling for better stability?

c) CT

d) Arrange occupational therapy for back door safety

a young girl had 1 week history of depressed mood , 2 weeks back she had sexual assault.

A.acute stress reaction

B.PTSD

C. malingering disorder

X ray pic with man persistant dry cough, dyspnea . heard PSM murmur.x ray pic. Dx?

AfraTafreeh.com
Get more from: MplusX Qbank (https://member.mplusx.com/)
Facebook Page: (https://www.facebook.com/mplusxqbank/)

a) Pulmonary hypertention

b) Pulmonary TB

c) Rheumatic fever

overdosed on 25mg indapimide (diuretic dose)? Or something. Was admitted and not able to
sleep despite benzodiazepam. Nurse found her staring at ceiling and other times agitated. Cause:

a) dementia

b) depression

c) catatonia

1. scenario of a lady who become agitated and confused at shopping center for 1-2 mins. She has
h/o same attacks . and during attack she stares blankly, doesn’t respond to any command followed
by conusion. Sometime during conversation with relatives it happens and she doesn’t follow the
conversation. What is ur dx?
a) Panic attack
b) GAD
C) Complex partial seizures
d)PTSD

a child with red & swollen scrotum not tender no fever

A.idiopathic scrotal edema

B.hydrocele

C. varicocele

Ansà idiopathic scrotal oedema

pt with carcinoma of ceacum asking most likely symptom??

A.altered bowel habit

B.lethargy/ tiredness

C.black coloured stool

AfraTafreeh.com
Get more from: MplusX Qbank (https://member.mplusx.com/)
Facebook Page: (https://www.facebook.com/mplusxqbank/)

D.recurrent colicky abdominal pain

1. your in a rural area and 4 cases of trachoma come to you then you find out extra 20 how can
you treat acute indexes ? Apart from hand washing , what will u give?
b-oral azithromycin
c-doxycyclin
d-don’t do anything
e-penicillin

1. Old man with recurrent falls in nursing home, he is found to have many bruises in head, ECG
showed sinus tachy, multiple ventricular ectopics and ventricular hypertrophy (written), on
enam/thiazide combination, BP standing 150/90 standing and systolic 90 in sitting, what invx for
diagnosis?
D) holter monitoring
E) 24 hr BP
F) Repeated BP measurements with postural change
G) CT head

40 years old man complain from persistent abnormal thoughts that’s making him washing his
hands at least 10 times after touching the key or door.he is on SSRI but want to try non
pharmacological therapy to help him get rid of this thoughts

Teach him how to avoid touching the doors or key

Explain him that this thoughts is normal to relieve his anxiety

Refer him to insight oriented psychotherapy

Have psychological evaluation

Picture ask for Dx. (same option)

Doctor after night shift from emergency department went to bar. He found his colleagues there
and some are having illicit drugs. They have to go to hospital for duty 2 hours later. What will u
do?

AfraTafreeh.com
Get more from: MplusX Qbank (https://member.mplusx.com/)
Facebook Page: (https://www.facebook.com/mplusxqbank/)

a) Report to director of emergency department

b) Tell directly to those colleagues

c) Leave and said nothing

d) Inform APHRA

- Scleroderma hand pic. Asking Dx.

Child comes with excessive vomiting for several times with abdominal pain.No features of
diarrhea. Finger prick test for glucose is normal .What is the immediate danger to him?

A. Cerebral oedema

B. Hypoglycemia

C. Hypokalaemia

D. Hyperkalaemia

Answer – A

10. 24yr old male with dysphagia and history of asthma in childhood. Endoscopy showed
narrowing and inflammation of middle esophagus, biopsy showed eosinophils in mucosa. What is
the best treatment?

a. Oral Antibiotic

b. PPI

c. Budesonide

Answer – C

Old age pt presented with c/o epigastric burning nd regurgitation his investigations shows

followin result what will you do for most appropriate management? ( 1.oesophageal dilation

2 laproscopic cardiomyotomy

3 nifedipine

AfraTafreeh.com
Get more from: MplusX Qbank (https://member.mplusx.com/)
Facebook Page: (https://www.facebook.com/mplusxqbank/)

(exact pic)

Hereditary Hypercholesterolemia...which is found?

A)Corneal arcus

B)Tendon xanthoma

C)Periorbital xanthelesma

D)Fatty liver

Answer – B

Egyptian woman who doesn’t understand English have to do endoscopy procedure. Her daughter
can speak English fluently and husband understand the procedure well. Who should interpret?

a) Her daughter

b) Her husband

c) official interpreter

d) non medical staff from hospital

e) 2 outside general practitioners?

Patient with ECG of AF. But the vital signs are BP-80/60 and unstable. Which of the following is
the best management??

a- Digoxin

b- Verapamil

c- Cardioversion

d- Sotolol

e- Disepynamide

.Cystic fibrosis case, what will be the chance to have disease child if one of them is carrier?

AfraTafreeh.com
Get more from: MplusX Qbank (https://member.mplusx.com/)
Facebook Page: (https://www.facebook.com/mplusxqbank/)

a)0%

b)25%

c)50%

d)50% of all male & no female

cyst in anterior neck(pic given), no pain, lump for 6 wks had h/o Sx of oesophagus 10 years ago
and another surgery(can’t remember) 2 years ago. Dx

a. branchial cyst

b. epidermoid cyst

c. metastatic cancer

Woman with 2-month h/o affected mood, in customer service role, missing work, can’t be
bothered to go to work, gets teary eyed often, recently hit her car to the pole, not paying attention,
worries in the night about losing her job, does not listen to her favorite music in the car, married 1
month ago to a caring person.

a)Adjustment disorder

b)Dysthymia

c)Major depressive episode

d)Generalized anxiety disorder

e)Borderline Personality disorder

Patient with history of DVT, HTN with DM patient on warfarin, metformin, statin and then add
amiodarone, 2 days later suddenly got swelling at right thigh and become painful, redness and
Temp 37.8. Right thigh size is 4 cm larger than left. Diagnosis?

A. Rhabdomylosis

B. Hematoma

C. DVT

D. Cellulitis

E. Drug reaction

AfraTafreeh.com
Get more from: MplusX Qbank (https://member.mplusx.com/)
Facebook Page: (https://www.facebook.com/mplusxqbank/)

Couple came for counseling of Schizophrenia. Husband’s mother has long term Schizo. He is
around 25 yrs old and is completely normal till now. No symptoms in the wife. What are the
chances of Schizo in the child?

a. 10 fold increase in risk

b. More than 20 fold increase in risk

c. There is no such risk in the child

d. There is an increased chance of Schizo spectrum disorders in the child

30-year old lady came for regular antenatal visit at 36 or 38 weeks of gestation (don't remember
exactly). She had no problems and everything was normal except a foetal bradycardia of 60/min.
An emergency c section was done but the infant still had the bradycardia of 60/min. What is the
cause?

A. anticardiolipin

B. factor V Leiden

C. Protein C

D. Lupus anticoagulant

E.anti Ro

53.

A little less redder than this pic ^^ Tx

a) prednisolone

b) timolol

AfraTafreeh.com
Get more from: MplusX Qbank (https://member.mplusx.com/)
Facebook Page: (https://www.facebook.com/mplusxqbank/)

c) chloramphenicol

. Malaria senario. . Asking which of the following most likely to exclude malaria??

A.Thrombocytopenia

B.daily fever

C.splenomegaly

58yr od male pt, since six month watery diarrhea mucus or blood ,with 3 kg weight loss.presented
with a hx of increasing difficulty to stand from sitting And walk upstairs. What is the cause?

a) Thyrotoxicosis

b) IBD

c) celiec

d) campylobacter jujeni

e) bowel cancer

59. A 7 years old girl with soiling in her underwear, she tells lies about accidents at school and
hide her soiled underpants and cry and quarrels with her brother when he calls her smelly. What
is your diagnosis?

a)Regression

b)Conduct disorder

c)Oppositional Defiant Disorder

d)Delayed milestones

e)Depression

58. an infant presented with a discharge from the umbilicus. On cleaning a red granulomatous
lesion was see, treatment

a. topical silver nitrate

b. surgical excision

AfraTafreeh.com
Get more from: MplusX Qbank (https://member.mplusx.com/)
Facebook Page: (https://www.facebook.com/mplusxqbank/)

c. steroid ointement

d. antibiotic

59. . diverticulitis scenario. Ct was given. What next

a. antibiotic and bowel rest

b. surgical exploration

c. colonoscopy

d. laparoscopy and proceed

60. a middle aged female presents with lt wrist pain after colliding her car with a pole. On
examination her lt pupil was constricted and also had decreased field of vision which she was
unaware of. What is the next app in?

a. ct head

b. iop measurement

c. slit lamp examination

61.Old woman 75 years of age is admitted to the hospital following a community acquired
pneumonia. She received antibiotics and is well on the 5th day. She suddenly developed rigors,
chills and high grade fever on the 4th day. What is the most likely explanation?

a. Hospital acquired pneumonia

b. IV cannula related bacteremia

c. empyema

d. Pulmonary embolism

62.Heart failure case man with AF on Ramipril, metoprolol, digoxin. Went on trip for 2 weeks
and stopped his medication. Now with bilateral oedema up to knee in legs. And ankle swellings
Lungs is clear. And the patient is not dyspneic, What is the most appropriate management?

A. Serum creatinine and elecrtrolyte

B. Echo

AfraTafreeh.com
Get more from: MplusX Qbank (https://member.mplusx.com/)
Facebook Page: (https://www.facebook.com/mplusxqbank/)

63. 17.Family with a newborn baby moved to a small village in rural Australia, father called up
GP to inform wife sad all the time, low mood, crying but taking good care of baby, cannot get her
to the hospital as he is busy with new work, what next:

A. Make a home visit

B. Send a social worker to check on woman

C. Insist husband to get the woman & baby to clinic

D. Admit the woman, baby can be with dad

E. Wait for the husband to call back again

8 yr boy with redness on scrotum, perinium n extending to penis no fever askin diag

A. idiopathic scrotal edema

B. torsion of testies

C. epididmorchitis

A patient has surgery and on 10th post op day she developed dyspnea & agitated, he being a
heavy drinker and a smoker. What is the next appropriate thing to do?

A. ABGA

B. Chest Xray

C. Blood alcohol

D. CTPA

E. Ultrasound

Answer – D

2. A patient on Thyroxine. TSH is low. What investigation to do?

CT scan

TSH and T4

AfraTafreeh.com
Get more from: MplusX Qbank (https://member.mplusx.com/)
Facebook Page: (https://www.facebook.com/mplusxqbank/)

Glucose

Electrolyte and cr

Mother with 2 pre-school children said that ‘she doesn’t want to take care of her children
anymore’ . what to do?

a) Report to child protective service

b) Arrange mother for psychiatrist referral

c) ??

Man came to psychiatrist for …. His examination are all normal. He has history of criminal
record for assault , theft etc . After counselling section, he confided to the doctor that he wants to
kill his ex-partner. What next?

a) Warn his ex-partner about it

b) Send him for forensic assessment

c) 3-4 continuous councelling session with him

A 6 years old boy contact with a chicken pox friend in school, he is neither vaccinated nor
infected previously, first step of mx

a. check serology

b. immunoglobulin

c. give vaccine to the child

25 years old man came to ask for the advice about the PSA screening in Prostate Malignancy as
his grandfather diagnosed at the age of 70 and his neighbor now diagnosed to have as he is now
65.What information will u give to him?

a.Screening is not indicated in this age

b.DRE is the good testing than PSA in this age

c.Can do now

AfraTafreeh.com
Get more from: MplusX Qbank (https://member.mplusx.com/)
Facebook Page: (https://www.facebook.com/mplusxqbank/)

d.It has high false positive rate

55 years man whose friend has prostate CA. ask for screening. His DRE is normal. What next?

a) Serum PSA

b) TRUS

c) Come back when symptoms appear

Man with surgery for clerk 1 metastatic melanoma , excise about 2mm margin of normal cell.

Malignant cell is around 1.5mm. He doesn’t have any other symptoms. Mx?

a) Regular observation

) Axillary LN biopsy

80 yr old lady with features of carcinoma of caecum.What would be the most easily found
symptoms??

A. Tiredness,lassitude,fatigue

B. RIF mass

C. Altered bowel habit

D. Bleeding PR

E. Melaena

Answer – A

96.Child,post viral ITP. Non blanchable petechial rashes. No other abn finding or bleeding. On
FBE Hb n WBC normal. PLATS 35×10 9. Treatment?

A.Strict bed rest

AfraTafreeh.com
Get more from: MplusX Qbank (https://member.mplusx.com/)
Facebook Page: (https://www.facebook.com/mplusxqbank/)

B.Steroids

C.Immunoglobulins

Answer – A (bed rest or observation)

Eye completely closed with redness and fever, O/E difficulty in eye movements asking for
investigation

A. CT head

B. Opthalmoscopic examination

C. Eye swab

D. Blood culture

Woman has regular menses for 4-6 wk interval, pain on left iliac fossa for 3 days, aggravated
while urination, her lmp was 9 days back, she had similar episode 4 wks back, which lasted for 3
days, cause?

a. Ovulation

b. complicated ovarian cyst

c. dysmenorrhea

d. early PID

Answer – A

102. Adrenal mass 5cm,incidental. Next?

A.Surgery

B.Observe

C.biopsy

Answer – A

The most common long term complication of streptococus pneumonia meningitis ?

AfraTafreeh.com
Get more from: MplusX Qbank (https://member.mplusx.com/)
Facebook Page: (https://www.facebook.com/mplusxqbank/)

a) cerebral palsy

b) deafness

c) epilepsy

Answer – B

An ambulance brought you a young lady who lost consciousness with body temperature
of 40° (hyperthermia) and BP was 155/90 when they found her collapsed at a friend’s birthday
party.
What is next in Mx?
Naloxone
Benzodiapines
IV Normal Saline.
Cold blanket and Cool N saline

Old lady with mild dermentia , took medicines regularly. Now, she said she doesn’t want to take
medicine . His son also told that she seems well without taking her meds. What to do?

a) Accept patient’s wish

b) Send medical staff daily to give her meds

c) Warn her that she cant go home if she doesn’t take these meds

Old recall of son dont wanna take care of her demented father anymore

Social review(exact word)

Discharge with his son and review later

Immediate arrange nursing care support

Patient who have performed IVF came for follow-up. How can know if it is successful? (sth like
that)

AfraTafreeh.com
Get more from: MplusX Qbank (https://member.mplusx.com/)
Facebook Page: (https://www.facebook.com/mplusxqbank/)

a) Transvaginal USG

b) Perform APPA?

c) Measure B hcg

Young woman comes to you complaining of heavy menstrual bleeding with pain. She describes
that clots of blood are lost in the first few days of menses with severe back pain. You ask her for
some investigations that need to be performed. She gets irriated and says she just wants a
treatment. Which of the following is the apporpraite next treatment ?
a. Tranexemic acid from day 1 of menses
b. Mefanimic acid from day 1 of menses
c. Mirena
d. Oral contraceptive pills

9. Girl (age not mention) stay with her foster mother. She is normally shy and not talk much but
lately she has unusual sexual behavior with her neighbor (sth like that). Mother is upset and
decide to send her back to her original foster care. Management?

a) Examine the general development of the girl

b) Referral to child psychiatrist

c) Arrange to send her back to foster

d) Arrange the case conference with mother and child proctective service

I got 2 questions about SVT

2 STEMI NSTEMI ecg

Onur yusl file


23-6-18

1- 6- month-old baby girl. Preterm 32 weeks-old born. Come to routine follow up. mother concerns
about her weight.Follow up weights :
Birth:1760 gr ; 1 month:2.2 kg ; 2 months: 3 something ; 3 months:smt ; 4 months: smt ; 5
months:smt ;6 months:6 kg
According the growth chart what should you say to mother ?

AfraTafreeh.com
Get more from: MplusX Qbank (https://member.mplusx.com/)
Facebook Page: (https://www.facebook.com/mplusxqbank/)

A- Her growth is slow but not delayed


B-Her growth is delayed
C-Her growth is between normal rang

2-67-year-old woman comes to your Gp clinic to get her influenza shot ,she had her pneumococal vaccine
3 years ago. her daughter is pregnant and the patient plans to visit her after delivery. apart from her
influenza vacc. what would be appropriate vacc for her ?
A-H.influenza
B-DTPa
C-Meningococ
D-Varicella zoster

3-In 100 people population 6 of them under risk of developing MI. This risk 50% more in smokers. If
there would be a new medicine which reduce the MI risk 1/3. The person who is smoker and use the the
new medicine what is the risk of MI for him ?
A-4%
B-6%
C-10%
D-12%

AfraTafreeh.com
Get more from: MplusX Qbank (https://member.mplusx.com/)
Facebook Page: (https://www.facebook.com/mplusxqbank/)

E-16%

4-19-year-old primigravida woman 35 weeks of gestation USG show total breech position. What to do
next ?
A-Normal vaginal delivery at term
B-ECV now
C-ECV after 2 weeks
D-c-section at term

5-70 year-ol male pt h/o DM, Hypertension, Coronary artery disease , Chronic AF. 2 weeks ago pt
admitted to hospital due to SOB and syncope and discharged with amlodipin.(Previous admission hx is
that short. No more details). Now patient suffers from righ thigh swelling. Righ thigh is red tender and 4
cm bigger than left thigh. fever :37,8 C ; pulse:90/min RR:14/min BP: 160/90 what is the cause of his
swelling ?
A-DVT
B-Cellulitis
C-Haematoma

6-8-year-old boy has left ear pain. O/E He has 39.1 C fever both left and right tympanic membranes are
bulging and there are suppurative tonsils. what is the diagnosis ?
A-External otitis
B-Otitis media

AfraTafreeh.com
Get more from: MplusX Qbank (https://member.mplusx.com/)
Facebook Page: (https://www.facebook.com/mplusxqbank/)

C-Tonsillitis
D-Mastoiditis

7-35-year-old woman had a serious migraine attack with weakness and numbness of left side limbs and
homonymous hemianopsia after 4 weeks she recovered but her vision still bad.she says she has to drive
her daughter to school and she asks you when to drive ?
A-She can drive now
B-After 4 weeks retest her vision and she can drive
C-She can drive only daylight not after sunset
D-She will never be able to drive again(amedex ans)

8-There was a picture of man who has a lump on his anterior triangle of neck, fine needle aspiration
biopsy shows malignant cells which one of the imaging leads to diagnosis?
A-X ray of head and neck
B-Ct of head and neck
C-MRI of head and neck

9-28-year-old man comes to ED after arguing with his girlfriend,her girlfriend wanted to break up with
him. He threatened her to kill himself if she breaks up with him. He doesn’t want to talk with you and asks
for a specialist ,he thinks you can’t help him. He doesn’t want to talk with any nurses as well and he
demands to leave immediately. What kind of personality disorder has he ?
A-Borderline
B-Histrionic
C-Narcissistic
D-Obsessive-Compulsive

10-You are a Gp at a rural area .In your town many cases occurred correlated with heavy metal poisoning
after a something factory starting up . What should you do ?
A-inform the city council
B-Take blood samples for heavy metal poisoning
C-inform the Public Health Service
D-inform the local media

June 23
1. Neighbourgh brought woman to GP. Woman is sayying that she is victim of domestic violence
and her 2 children saw them beating her. She is afraid to go back home. What should you do?
a. Ask neighbor to drive her back home
b. Inform police
c. National child service
d. Talk to husband
e. Contact domestic abuse and violence centre
2. A CT brain of cerebral haemorrhage. A patient is taking warfarin for AF, his INR 2.9, Mx?
A. stop warfarin
B. give FFP
C. vit K
D. PRP
E. packed cell
3. Female admitted, overdosed on indapamid , was admitted and not able to sleep despite
benzodiazepam . Now nurse found her staring at the ceiling and other times agitated. Cause?
A. Dementia
B. Delirium

AfraTafreeh.com
Get more from: MplusX Qbank (https://member.mplusx.com/)
Facebook Page: (https://www.facebook.com/mplusxqbank/)

C. Depression
D. Catatonia
4. 68years old man who is living alone, taking perindopril, B blocker diuretics for CCF,
Elderly care service visit him once 2 weeks for medical checkup. He walks every day for 30
min and does his own shopping. What is the greatest risk factor for fall in this man?
a. health care service visit him not frequent enough
b. taking more than three drugs
c. living alone
d. health care service being too far away from him
5. 45 year old patient comes to you fasting blood sugar is normal , he asks you when should do
screening for blood sugar ?
• FBS yearly
• OGTT every 2 years
• Random blood sugar every 2 years
• HBA1C every 12 months
6. There was a question about Forest Plot. Looks like this but the numbers at the base are 0.001,
0.01, 1, 10, 100. They conducted a study about . The graph measures the presence and
absence of pain. What does the plot mean?
a. DRUG X is more effective than Drug Y
b. DRUG Y is more effective than Drug X
c. Forgot
7. ***long scenario(looks like MND)man comes with clumsiness of hands,fassciculations,proximal
muscles weekness,foot drop,ix?
a. EMG
b. MRI spine
8. Stat asirin question? 2%
9. Patient’s RPR 6 months back was 1:200, now its 1:2. antibodies were positive 6 months back
and now.
a. Patient is syphilis carrier
b. Patient has syphilis now
c. Patient doesn’t have syphilis now
10. Most teratogenic for fetus in pregnant lady?
A) cocaine
B) heroin
C) amphetamine
D) methamphetamine

AfraTafreeh.com
Get more from: MplusX Qbank (https://member.mplusx.com/)
Facebook Page: (https://www.facebook.com/mplusxqbank/)

E) Flouxetine

11. History of appendicetomy & cholecystectomy presented with 3 wk history of abd pain,
distension, BS exaggerated , CT abd image given
a. Sigmoid volvulous
b. Adhesive IO
c. CA Sigmoid
d. Pseudo obstruction
12. Old man 75 years of age has 3 week history of constipation. He presents with mild abdominal
tenderness and on rectal examination there is no faeces in the rectum. On CT there is cut off
at the sigmoid colon and dilation of the ascending, transverse and descending colon. Which of
the following is the most likely diagnosis? (no ct given)

a. Sigmoid volvulus

b. Colon cancer
c. Caecal volvulus

13. A patient with no family history of DM came and ask you for diabetic screening, what should
you do?
a. Random blood sugar now
b. FBS every 3 years
c. OGTT now
14. 2. Scenario of a child who is going to camp had asthma, teacher call up and ask what to do
because unable to contact the parents
a. Ask the teacher to bring the child to see you
b. Fax teacher the asthma action plan
c. Try to contact the parents
15. Stress incontinence scenario on cough or sneezing ....best next invest,
Ct
Urodynamic studies
Urine analysis
Cystoscopy

AfraTafreeh.com
Get more from: MplusX Qbank (https://member.mplusx.com/)
Facebook Page: (https://www.facebook.com/mplusxqbank/)

1st investigations to order empty supine stress test urinalysis postvoid residual measurement (PVR)
cough stress test Full details Investigations to consider pad test urodynamic testing Q-tip test
cystourethroscopy Full details

16. Baby 6 hours of delivery develops jaundice cause?


Neonatal sepsis
Hemolytic anamia
Breastfeeding
17. A child had jaundice on the 3rd after birth, now baby is 6weeks and still having jaundice. What
is the cause?
a. Gilbert’s disease b. Biliary atresia c. Physiologic jaundice
18. 7 days old baby presented with jaundice since 4 days of life, his birth was at term, not
complicated and he is breast feeding , serum bilirubin level was 240 (normal <200) with 120
direct bilirubin (it’s 120 not 12 confirmed), his liver is 1 cm palpable below costal margin. What
is the most appropriate next step?
A-thyroid function test
B-abdominal ultrasound for liver and biliary tree.
C-coombs test
D-stop breast-feeding and follow up
E-phototherapy
19. 55 yr old man presents with three week history of nausea & vomiting! Initially clear fluid, later
indigested food particles! Last 3 months, he suffered from abdominal pain radiating to back!
Most likely Dx? This is full scenario, no jaundice in scenario.
1. Ca head of pancreas
2. Chronic DU
3. Ca duodenum
4. Ca body of stomach 5. Linitis plastica
20. 2 yrs went to some country 6 months ago, penile ulcer, healed in one week, now generalized
lymphadenopathy, rash on trunk and limbs young homosexual male dx?
a.HSV
b.HIV
c.treponema pallidum
21. Pt with deep jaundice moderate ascites confused complaining of abdominal pain flapping
tremors hepatic enceph features.liver difficult to palpate.high fever 38c.what is next
appropriate step?
A.abdominal paracentesis
B.i/v antibiotics
C.blood culture
D.fluid restriction
E.usg
22. 9. Young female schizophrenic patient in psychiatric ward suffers ulcer in right arm, you where
informed by the nurse that the patient will not accept any intervention for this ulcer. What will
be your action ?
A call her parents for consent
B obtain medical board power order to treat the patient
C refer to surgical team
D take a swab
E neglect the ulcer
24. 9. Young female schizophrenic patient in psychiatric ward suffers ulcer in right arm, you where
informed by the nurse that the patient will not accept any intervention for this ulcer. What will
be your action ?
A call her parents for consent
B obtain medical board power order to treat the patient
C refer to surgical team
D take a swab
E neglect the ulcer
25.

AfraTafreeh.com
Get more from: MplusX Qbank (https://member.mplusx.com/)
Facebook Page: (https://www.facebook.com/mplusxqbank/)

26. pic of old lady of facial palsy. loss of sensation in the left site of the face,pain in left ear &
discharge (no vesicle in ear in the scenario).asking tx??
a.prednisolone
b.acyclovir
c.antibiotic
27. 93 year female who is getting treatment from you for osteoarthritis. Now she is on Paracetamol
1000mg 3 times daily and Paracetamol 500mg+ Codeine at night for pain. She said as she
was not getting relief with this, her friend used to inject her morphine, which was given for her
palliative care. Now the friend died and she wants you to prescribe morphine for her. What is
the best step?
A. Prescribe her morphine.
B. Refuse her request.
C. Refer her to an Orthopedic surgeon.
D. dose of paracetamol Increase the
E. Inform authorities about her drug seeking behavior

28. 72 year female come for routine check up. Her mother develop colon cancer at the age of 65
years. She doesn’t have any bowel symptoms and haven’t done any screening test before.
She want to know her risk?
a) Colposcopy 2 yrly
b) 5 yrly cervical cancer screening
c) 5 yearly colon cancer screening
d) mammogram year
e) no screening needed
29.
30. 4 years old girl. Mother notice purulent amount of blood in her underwear for 10 days. On
examination , there is small amount of blood stained in vulva. What is ur first step in the
management? Not developed any pubic hair and no breast development.
a) Report child protective service immediately for sexual abuse.
b) Microsopic blood culture of the Swab from the vulva
c) oral Amoxil (not sure)
31. 3. 52 years old woman with incontinence of urine during coughing and laughing. Her
urodynamic study shows that her incontinence occurs during low volume study. On
examination she had mild rectocele. Her symptoms showed some improvement after pelvic
floor exercise but she could not satisfied this result. What will your appropriate step
management?
A-Bladder neck suspension
B-Topical Estrogen cream
C-Weighted vaginal cones
Do not remember
32. 4. Men 35 years with history of difficulty in micturition for 2 years. Difficult to start and dribbling
at the end of micturition.
a) BPH
b) Bladder stone
c) Urethral stricture
d) Urethral valve
e) Cancer (not sure)
33. 5. Old man with urinary incontinence. Not severe but it causes a problem when he goes out.
First step of Mx?
a) Pelvic floor exercise
b) Bladder train
c) Prazosin
d) Anticholinergic agent
34. 6. Women with UTI and she suffer from significant persistent pain. She is also depressed,
insomnia , low mood, etc. Mx?

AfraTafreeh.com
Get more from: MplusX Qbank (https://member.mplusx.com/)
Facebook Page: (https://www.facebook.com/mplusxqbank/)

a) Tramadol
b) Diazepam
c) Citalopram
d) Diclofenac
If mirtazapine then that
35. 7. Diabetes ulcer on the foot pic.
diabetic pt, non healing ulcer, pic 2 ulcers in medial malleolus, and foot pulses not pulpable. X
ray already done. what to do for Mx?

A) blood culture
B) white cell scan
C) swab from ulcer
D)venous dupplex USG
E) MRI foot
36. Woman 55 years came to doctor for regular medical check up. Her mother has CA cervix at 63
years. Maternal aunt has breast CA at --- year . and father has colon cancer at 72 years. What
to check her?
a) Colposcopy 2 yrly
b) 5 yrly cervical cancer screening
c) 5 yearly colon cancer screening
d) mammogram yearly

37. 12 week pregnant with lower abdominal pain. Normal nuchal usd at 11wk. Heavy PV bleeding,
BP 80/50
incomplete miscarriage
Rupture ectopic
Missed miscarriage.

INDIA- 23rd June 2018


Overall exam was alright. A lot of recalls, no ecg, no cardiac questions, no fundoscopy,
new questions mentioned separately.

NEW QUESTIONS:

1) A 41 yr obese female, pregnant 32 weeks gestation, presents with upper abdominal


pain since a few days, bp-150/92. Occasional headache. All other examination- WNL.
What is the diagnosis?
a) Acute cholecystitis
b) Pre eclampsia
c) Concealed placental abruption

2) Not sure about age, obese scavandian female for first antenatal check up(@ 8 weeks) .
What will you do ?
a) OGTT now
b) OGTT @ 24-28 weeks
c) RBS @28weeks

AfraTafreeh.com
Get more from: MplusX Qbank (https://member.mplusx.com/)
Facebook Page: (https://www.facebook.com/mplusxqbank/)

3) Your female patient calls you, says husband is getting aggressive and violent. She is
hiding in bathroom while calling you and you can hear husband shouting in
background, what will be your initial appropriate advise?
a) Ask her to go to women’s shelter
b) Encourage her call the police immediately

4) You are a rural GP. You are known to both husband and wife and have good social
relationships with them. At a gathering, Wife secretly and tearfully confesses to you
that her marriage is not working. What will be your action?
a) Arrange appointment for a long interview
b) Arrange a couple interview
c) Send her to another GP
d) Console and embrace her now

5) A middle aged man comes in ED saying something about danger to his life foe which
he has kept a knife with him, although he know he shouldn’t. on asking, he evades
about weapons. And refuses you to contact his psychiatrist whom he saw last week.
What will you do?
a) Contact psychiatrist against his will
b) Wait until he agrees to see his psychiatrist(not sure about exact words)
c) Observe
d) Other irrelevant options

6) Old age man case of metastatic pancreatic carcinoma. Complaints of pain, lethargy
and constipation. Lives alone, daughter makes a visit most of the evenings. He is sad
and hopeless. What is most important thing to access immediately in this patient?
a) His pain score assessment
b) His suicidal thoughts
c) Any drug abusive behavior
d) Others irrelevant

7) You are a member of an infection control team of your hospital, you found out
increased infection rate for one surgeon with same procedures and surgeries. What will
be your most appropriate action?
a) Talk to the surgeon
b) Talk to the head of surgery unit
c) Discuss with your infection control team
d) Inform the director

8) You remember that you prescribed pantoprazole to wrong patient yesterday. Now
patient is stable, no fresh complaint. What should be your most appropriate action?

AfraTafreeh.com
Get more from: MplusX Qbank (https://member.mplusx.com/)
Facebook Page: (https://www.facebook.com/mplusxqbank/)

a) Inform patient that you made a mistake


b) Tell patient that this is drug for acidity
c) Do nothing
d) Others I forgot

9) 9 months old child presented with abdominal distension, elder sibling had h/o RTI few
days back. Abdominal xray- paucity of air in right quadrant. What is the most
appropriate treatment?
a) Air enema
b) Iv antibiotics
c) Others I forgot

10) Elderly diabetic, what to check 3 monthly?


a) FBS
b) HBa1c
c) Fasting lipid profile
d) ACR

11) DKA scenario 14 years old male, what to check immediately in ED to make diagnosis?
a) Urine ketones
b) USG abdomen
c) ABG
d) CT head

12) A young diabetic patient comes with h/o one episode of hypoglycemic attack after
gymming for few hours. Never had any episode before or after that. Asks advise for
driving now?
a) Cant drive for 6 weeks
b) Cant drive for ?1year
c) Should avoid drive after vigorous exercise
d) Should drive only after fasting glucose levels maintained @3mmol/l

13) One night you see your patient driving his car , when he is not allowed to drive at
night. And you are sure that it was your patient, not anyone else. What will be your
most appropriate step?
a) Inform to driving license authority without knowledge of patient
b) Invite patient to discuss with him first
c) Ignore

AfraTafreeh.com
Get more from: MplusX Qbank (https://member.mplusx.com/)
Facebook Page: (https://www.facebook.com/mplusxqbank/)

14) MND scenario with mixed features of right upper limb and left lower limb. What is
most appropriate investigation?
a) EMG
b) MRI cervical spine
c) Nerve biopsy
d) ACE receptor antibodies

15) Middle aged man presents with pain from sole to great toe of right foot, which awakes
him from his sleep. O/E: inflammation from sole to great toe, non tender, active and
passive movements are painless and free. No sensory loss, SLR- positive @30 degree.
Pedal pulses normal. No h/o dm/gout/arthritis. Reflexes are normal. What
investigation is most important for diagnosis?
a) Serum urate
b) MRI lumbar spine
c) Others I forgot

16) Old diabetic patient , mentioned well controlled diabetes upto last week. Now presents
with (pic given- two infective ulcers @medial malleolus of left foot). Pulses normally
palpable, no sensory deficit. What will your next step after cultures and antibiotics?
a) Plain Xray foot
b) MRI foot
c) Arteriography
d) Duplex doppler of lower limb

17) Ques with this foster plot telling about meta analysis for comparison between
antibiotic X and Y. what is your interpretation?
a) Antibiotic X is more effective than Y
b) Antibiotic Y is more effective than X
c) data is insufficient
d) Meta analysis not adequate to make any conclusion

AfraTafreeh.com
Get more from: MplusX Qbank (https://member.mplusx.com/)
Facebook Page: (https://www.facebook.com/mplusxqbank/)

18) PAP smear report: intermediate risk, HPV(not 16,18:mentioned) positive and LSIL
present. What do to next?
a) PAP smear after 12 months
b) Colposcopy within 3 months
c) HPV after 12 months
d) Cone biopsy within 12 months

19) Despite increased no of patients covered under cervical cancer screening (from68% to
85% now). There is no decrease in cervical cancer incidence. What is the cause?
a) Screening test is not specific
b) Cervical cancer has very rapid course
c) Treatment not adequate

20) Father brings her girl child in ED , now father doesn’t want to leave her in ED , what
to do?
a) Discharge and send her home with father
b) Ask if she needs her father to be with her in ED
c) Sorry forgot other options

21) Your colleague who is anxious and insomniac due to some board exam, asks for
sleeping pill prescription from you. What will be your most appropriate action?
a) Advise regular exercise and relaxation techniques
b) Prescribe him according to his wish
c) Ask him to consult his physician
d) Prescribe him melatonin agents as they have better efficacy

AfraTafreeh.com
Get more from: MplusX Qbank (https://member.mplusx.com/)
Facebook Page: (https://www.facebook.com/mplusxqbank/)

22) Patient TFT given: TSH on higher side, T3- low, what next investigation to do?
a) MRI brain
b) Thyroid antibodies
c) Thyroid scan
d) USG neck

23) Patient present with c/o shooting pain along the back of the leg and sole of foot. What
additional finding helps to localize the lesion?
a) Loss of ankle jerk
b) Loss of knee jerk
c) Loss of sensation on medial foot
d) Loss of sensation on medial leg

24) Woman comes to see a male physician. She need to get pelvic examination but she is
reluctant to get it done from male doctor, what should be your appropriate action?
a) Ask her if presence of female nurse would help
b) Ask her to come when female physician available
c) Send her to some another physician
d) Others irrelevant

25) Police brings in a drink and drive case, agitated, with blood alcohol level- 0.2%. what
will be your action to take appropriate history?
a) Observe until he gets stable to give history
b) Other options I forgot

26) Female forgetful and keep losing track of time, what to check?
a) Orientation
b) Insight
c) Constructional apraxia
d) Delusions

OLD RECALLS: just copy pasted from old recalls files.

27) Child with impetigo. Doctor started antibiotic and covered the lesions with dressing,
when he can go back to pre school?
a) Now
b) After lesions dry

AfraTafreeh.com
Get more from: MplusX Qbank (https://member.mplusx.com/)
Facebook Page: (https://www.facebook.com/mplusxqbank/)

c) After 2 days of antibiotics

28) Couple come for infertility problem for the last 12 months. On testing examination
and tests of the female are unremarkable. Male has azospermia. And bilateral absence
of vas deferens. Which of the following is most important test before the starting the
treatment of infertility?
a) No testing required as they cannot have a child
b) Testing of both male and female for cystic fibrosis
c) Refer for IVF
d) Serum FSH and LH for male
e) Serum Testosterone level

29) 8 yrs old with type 1 diabetes on insulin glargine. Every morning glucose levels is
high. What to do ?
a) check blood glucose levels at 2:00-3:00 am for 3 days
b) check early morning insulin levels
c) increase the evening intermediate insulin
d) give insulin before breakfast
e) give another dose before sleep

30) Peptic ulcer case with Hpylori positive, took triple therapy (Amoxil+metro+PPI) for
a week or 10 days(not sure). Urea breathe test still positive after 6 weeks. Asking the
reason?
a) Resistant to metronidazole
b) Resistant to amoxil
c) Not reliable urea breath test
d) review in next week

31) Mother brings her 7 months old son with complaints of bleeding from mouth and
gums. she told you that he bumped his head somewhere so that he presented like this,
on examination the child was bleeding from frenulum, old bruises on shins and
forehead. No hepatosplenomegaly. No petechial rash. What is the most likely
diagnosis?
a) factor 8 deficiency
b) non-accidental injury
c) Von willebrand disease
d) leukemia

32) 92-old patient found unconscious at home brought by son to emergency department .
the
patient was taking tablets methadone, oxycodone , others , patient has constricted
pupil on examination which of the followings is the likely cause?

AfraTafreeh.com
Get more from: MplusX Qbank (https://member.mplusx.com/)
Facebook Page: (https://www.facebook.com/mplusxqbank/)

a) methadone
b) oxycodone
c) may be cocaine don not remember other options

33) Woman 51 years of age presents after menopause for the last 8 months. She complains
of
irritability, moodiness, low libido and hot flushes. Which of the following would be
most appropriate to give to this woman?
a) Combined continuous estrogen and progesterone throughout the cycle
b) Continuous estrogen throughout the cycle
c) Continuous estrogen with progesterone first 12 days of the cycle
d) Continuous progesterone with testosterone

34) A man in brought to the ER after brawl hit to the side of left eye. On examination,
you have found an orbital floor fracture. Which of the following is the accurate
predictor of this diagnosis?
a) Sub conjunctival haemorrhage
b) inability to open his mouth
c) loss of sensation of his cheek
d) decreased visual acuity

35) An elderly woman on polypharmacy GFR was 6 weeks back came with dyspepsia, was
put on Olazpine and anatacids and is now presenting with confusion, loss of appetitie
and nausea.􀳦Sodium 129, k 5.3, calcium 3, phosphorus high , creatinine high, urea
very high and bicarbonate 20, what is the reason for the confusion?
a) Uremia
b) hyponatremia
c) hyperk
d) hyper ca
e) metabolic acidosis

36) post partum episiotomy wound- keep the wound clean.

37) Old recall of patient with c/o claudication and diminished pulses, smoker and unfit for
surgery, what is the most appropriate advise?
a) Reduce smoking
b) Supervised exercise
c) Rest irrelevant

38) Screening of 52 years old with positive family h/o colon cancer @58 years?
a) FOBT annually

AfraTafreeh.com
Get more from: MplusX Qbank (https://member.mplusx.com/)
Facebook Page: (https://www.facebook.com/mplusxqbank/)

b) Colonoscopy annually

39) Screening for diabetes with no family history?


a) FBS yearly
b) FBS 3 yrly
c) RBS 3 yrly

40) Aspirin recall: 100% risk in those not taking aspirin

41) Best study to check effect and efficacy of drug?


a) Systemic review
b) Rct?//
c)
42) Locals of one area affected by coal seam. Physical examination- WNL. What will be
your most appropriate action?
a) Inform public health unit
b) Educate about environmental risk
c) Do other tests of patients

43) That recall of BPD with BP cough around the neck. What is the diagnosis?
a) True suicidal attempt
b) Acting out
c) Displacement
d) Others I forget

44) Decreased appetite BMI- 12 young girl, what to ask in history?


a) Feeling of inferiority

45) Investigation of down syndrome @ 15 weeks?


a) Amniocentesis

46) The famous ‘mamu’ question

47) Borderline personality disorder, treatment?


a) DBT

48) 6 months child with vaccination only at birth, nothing after that. Now presents with
URTI. What will you do?

AfraTafreeh.com
Get more from: MplusX Qbank (https://member.mplusx.com/)
Facebook Page: (https://www.facebook.com/mplusxqbank/)

a) Give vaccination and catch up schedule

49) Young patient notices mass in the groin on straining. USG showed fatty tissue
protruding from deep inguinal ring. What is the most appropriate management?
a) Reassurance
b) Groin sling
c) Open mesh repair
d) Laproscopic hernial repair

50) Best test to confirm CV infection in infants?


a) HCV serology @18 months

51) HCV + pregnant lady comes for antenatal check up and asks advice about risk of
transmission to fetus. What will you advise?
a) Prevent fetal scalp ph monitoring

52) a child 10 yr old refuses to go to school . he developed frequent touching of his face
and raise his arm above his head and crawl his hand on the ground, he can supreess
doing those thing s but developed anxiety if he suppress those things . his classmates
make fun of him coz of those behaviours and his teacher also annoyed,what is the
diagnosis?
a) asperger disease
b) ADHD
c) complex motor tic disease
d) Tourette syndrome
53) Young primigraivda delivered a healthy 3250gm baby after an uneventful labour
lasting only four hours. During labour, no analgesics or epidural was required. There
was slight meconium staining. The fetus developed severe central cyanosis, limp and
not breathing, HR-40/min. What is the most important next step to resuscitate the
baby?
a) Bag and mass ventilation
b) Intubation and ventilation
c) Intravenous sodium bicarbonate
d) Intravenous glucose
54) Pic of obese male patient and long tubuler elongatred mass in abdomen from
xiphisternum to umbilicus. Description given that mass becomes prominent when
patient raises his head, asking management?
a) Physiotherapy
b) Herniorhapphy with mesh repair
c) Reduce weight

AfraTafreeh.com
Get more from: MplusX Qbank (https://member.mplusx.com/)
Facebook Page: (https://www.facebook.com/mplusxqbank/)

d) Reassurance

55) Similar ques without pic, asking advice for patient? With similar options
56) Young age primi presented to you at 36 w ,she is telling you that her friends tell her
abdomen looks smaller ,she came at 32 w and everything was normal, usg at 32 weeks
-normal. What is the most important indicator of her baby's growth:
a) the fundal level is increasing in every visit
b) Her pubic fundal height of 34 cm at 35 weeks
c) 5 fetal movements in 12 hrs
d) normal CTGy

57) A policeman is coming for his colleague that beaten his friend during an argument, he
is intoxicated for alcohol what in history will be important to say personality disorder
a) History of fight with colleague two weeks ago
b) History of cruelty to animals in childhood
c) Alcohol and drug abuse
From amedex
58) Old age ,felt 4 hour retrosternal chest pain ,the pain was proceeded by aggressive
vomiting ,left lower lung dullness ..invx:
a) Troponin
b) Gastrografin contrast
c) CXR

59) Young female came with fatigue and tiredness ,Hb decreased from 11 to 9 Ca 1,9
INR 1.6
Na 130
K 5.5
Next best
a) blood transfusion
b) IV Ca
c) vit K
d) Na
e) Potassium

60) Man with nocturia n urinary frequency n dribbling. mass above the pubic symphysis
DRE showed enlarged prostate with palpable median sulcus palpable.what is the most
appropriate next step?
a) trans rectal ultrasound
b) PSA
c) bladder scan
d) CT
e) urine cytology

AfraTafreeh.com
Get more from: MplusX Qbank (https://member.mplusx.com/)
Facebook Page: (https://www.facebook.com/mplusxqbank/)

61) A child with fever and hx of sore throat 10 days back . He developed pain in wrist
later developed arthralgia n swelling in ankle joint. skin rash present . what's the most
initial step of management ?
a) ESR
b) Full blood examination
c) throat swab
d) USG
62) aboriginal child comes with purulent ear discharge for 6 weeks what will u do after
clearing the discharge
a) amoxicilin
b) ear toilet…if chronic >6 weeks
c) prednisolone
d) reassure
e) cipro ear drops

63) 75 year old man comes to you with progressive constipation for last 3 weeks with
absence of flatus. Abdominal distension is becoming worse. Mild tenderness is present.
He had history of cholecystectomy 25 years ago. Dilated bowel loops along ascending
transverse and descending colon. Rectal examination is empty. What is the diagnosis?
a) Adhesion obstruction
b) Sigmoid volvulus
c) Carcinoma of sigmoid colon
d) Ceacal volvulus
64) similar scenario with CT pelvis pic, similar options, diagnosis?

65) Day 4 passing of meconium stained stool, diagnosis?


a) Hirshpung’s disease

66) 5 year old child presents after sustaining lacerated wounds on leg, received 2 doses of
dtp at 2 and 4 months. What to give now after wound management?
a) TT and antibiotics
b) TT and Ig
c) DTPa and booster after 2 months
d) DTPa and IG

67) Mother of 10 weeks old child comes 5 th time in 10 days. Says child cries excessively
and is difficult to settle. She is able to breastfeed him. What in history will help to
reach diagnosis?
a) Her thoughts about child’s behavior
b) Her thoughts about child care skill
c) Her thoughts about harming the baby

68) 24 months child verbal milestone?


a) 2 pronouns

69) 80 years old women in a seperated ward,nurse says she found masturbating herself
frequently..what to do?-

AfraTafreeh.com
Get more from: MplusX Qbank (https://member.mplusx.com/)
Facebook Page: (https://www.facebook.com/mplusxqbank/)

involve social worker ,


tell her it is not good,
send her in a 4 patient room in ward
a) Tell nurse patient needs privacy
70) A lady brought by her friends , she feels irritable and frightened she feels anxious
outside and enjoys her life in her home , she does gardening and read books and do
crafting , what is the most significant in her history
a) Alcohol or drug abuse
b) school refusal
c) suicidal thoughts

71) A child presents with increasing fever and cough of 2 days duration. X-ray was given
which showed right sided pleural effusion. Which investigation aids in the definitive
diagnosis of the causative organism?
a) Blood culture
b) chest x-ray
c) pleural aspirate
d) pneumococcal PCR
e) Sputum culture
72) 25 year indigenous lady presented with sob and cough since 3 weeks. o/e temp 37.8,
PR, RR, BP, saturation all are within normal limits, mitral stenosis with bilateral
basal crepts. X-ray given with features like bilateral symmetrical upper lobe infiltrates
(X-ray seems like apical but infiltrates are not too high at the apex). Diagnosis?
a) Left ventricular failure
b) Pulmonary hypertension
c) rheumatic fever
d) Pneumocystis pneumonia

73) Woman feels she has worms in her stomach because of a recent volcano eruption in
indonesia. Previously on trifluorperazine but stopped due to hand stiffness. What to
give now?
a) quetiapine
b) Olanzapine
c) Recommence trifluorperazine
d) stop trifluoperazine

74) Gout scenario. With a man comes with acute symptoms. Creatinine level – slightly
increased above upper limit. What would be your most appropriate management??
a) Colchicine
b) prednisolone
c) NSAID
d) Allopurinol

75) 35 year old woman presented for screening, never in any screening program earlier. She
is otherwise fine. Her BMI is normal. All investigations were normal. What will you
do?
a) RBS every 2 years
b) OGTT every 2 years

AfraTafreeh.com
Get more from: MplusX Qbank (https://member.mplusx.com/)
Facebook Page: (https://www.facebook.com/mplusxqbank/)

c) Observe till age 45, then 3 yearly blood sugar


d) FBS every 3 years
e) FBS every 12 months
40 for DM screening.. 45 for hypercholesterolemi
76) A 32 year old lady presents to you asking for advice regarding the need for Chlamydia
screening. She tells you she has not been sexually active for the past 2 years since her
last partner, at which time she was screened negative for Chlamydia. She now inquires
about the reason why she should undergo a Chlamdyia screening test. Your answer
would be:
a) It can be asymptomatic
b) It causes infertility.
c) Her partner could be asymptomatic.
d) She does not need the test

77) 40 years old woman with menorrhagia and hysteroscopy. She had a dilatation and
curettage three months ago. Which of the following is the MOST appropriate
management plan?
a) Nortestosterone
b) Levonorgestrel iucd
c) continuous medroxyprogesterone
d) Tranexamic acid during period

78) child with URTI, protein+, rbc+ on urinalysis, came back 2wk later, urine, rbc+ of
non glomerular origin, no cast. Which investigation?
a) Urine culture
b) ASO titer
c) bladder scan
d) DMSA
e) Renal Biopsy

79) Young woman 26 years of age comes to you complaining of heavy menstrual bleeding
with pain. She describes that clots of blood are lost in the first few days of menses
with severe back pain. You ask her for some investigations that need to be performed.
She gets irriated and says she just wants a treatment. Which of the following is the
apporpraite next treatment ?
a) Tranexemic acid from day 1 of menses
b) Mefenimic acid from day 1 of menses
c) Mirena
d) Oral contraceptive pills
e) oral medroxyprogesterone acetate

80) complaining of soreness on lateral epicondyle at the end of work, the patient
attributes it due to computer mouse clicking.On examination , tenderness on lateral
epicondyle .Asking treatment?
a) lateral epicondylectomy
b) finger immobilization splint
c) lateral epicondyle immobilization splint

AfraTafreeh.com
Get more from: MplusX Qbank (https://member.mplusx.com/)
Facebook Page: (https://www.facebook.com/mplusxqbank/)

d) bracing under lateral epicondyle


e) oral steroid

81) MVA. CT scan (given)- hemopneumothorax on right side. He is in distress, shallow and
short breathing oxygen saturation -88%. What is your next appropriate management?
a) Morphine
b) Needle thoracotomy
c) Chest tube drainage

82) A sudanese boy had sex with his girlfriend. after a few days he found urethral
discharge from his penis. He has other sexual partners. what specimen will help you
make a diagnosis of the STI
a) 1st catch urine PCR
b) Mid stream urine pcr
c) first stream urine microscopy and culture
d) Urethral swab gram stain and culture
e) mid-stream urine microscopy and culture

83) You are intern. The patient is intubated and one of his family members says the
patient wants to change the will. What should you do?
a) Witness the will
b) Refuse to witness
c) Ask surgical register if he can do it
d) Seek legal advice
e) Ask the nearby kins and witness the will

84) 74 year old man brought from resident home, he is masturbating near nurses’ station,
tried to cuddle and kiss the nurses while bathing him. History of Alzheimer’s and is on
donepezil and moclobemide. Diagnosis?
a) Progressing alzeihmer’s
b) Side effect of donepezil
c) Side effect of moclobemide (MAO inhibitor)
d) Frontal lobe space occupying lesion
e) Dementia

85) Female patient with RA she takes ibuprofen and methotrexate to control her disease
for 5 years. Recently patient complains of some symptoms (jaundice), and her labs are
given.
protein - 9 (6-8) slight increased
albumin- 5 (3.5-5.5) normal
ALT - >100 (7-56) Liver enzymes >100 in my exam question
AST - >100 (10-40)
GGT - >100 (0-30)
What is the cause of her symptoms?
a) Methotrexate induced hepatotoxicity
b) ibuprofen induced hepatitis
c) Autoimmune hepatitis

AfraTafreeh.com
Get more from: MplusX Qbank (https://member.mplusx.com/)
Facebook Page: (https://www.facebook.com/mplusxqbank/)

d) Viral

86) low BMI girl 15 and she stopped going to Ballet classes , has fights with her father and
when you interviewed her she tells what's the need for all of these , what to do next :
a) liase with father
b) discuss safety plan for her and her family

87) a lady complains of headache for 8 months that awakes her every morningShe has
been taking paracetamol and OTC pain medication that relief her 1 or 2 hours and
then feel pain again she mainly complains that the headache in bitemporal OR frontal
a) Tension Headache
b) Drug rebound headache
c) Migraine

88) 15 y/o boy with h/o sudden outburst of anger who is currently in a special school is
now at verge of expulsion. What would be the best drug management for this case?
a) Sodium valproate
b) Risperidone
c) Olanzapine
d) Haloperidol
e) Carbamazepine

89) Farmer with hand deformity dupuytren contracture,(picture given) inability to extend
his ring finger, history of cut injury in that fingers (+). He takes 4 beers per day and
30 pack of smoking per year. What is the cause of his injury?
a) chronic infection
b) his farmer instrument
c) alcohol
d) Inherited disease

90) Infant with jaundice started on day 4 o with total bilirubin 240 & conjugated 120,
asking what most app to do?
a) Phototherapy
b) USG abdomen
c) Skip breast feeding for 2 days
if cong. bilirubin is high so it is biliary atresia and US sh. be done ..but if non conj. high so its mainly
breast milk J..so the answer will be C or B as stopping breast milk is not recommended unless for
diagnostic purpose

91) A case of spontaneous rupture of membranes at term.Now cervix is 8cm dilated and
head at ischial spine .CTG shows heart dropping to 70 for 3 minutes with baseline
heart rate 110bpm.What next?
a) Fetal scalp ph
b) Immediate C sec

AfraTafreeh.com
Get more from: MplusX Qbank (https://member.mplusx.com/)
Facebook Page: (https://www.facebook.com/mplusxqbank/)

c) Ventouse delivery
d) CTG
e) Stop syntocin

92) Man 1 week after prostatectomy, his wife came to consultation because she is
concerned her husband is acting different, more irritated, aggressive, shouting. what
in the history will help u reach diagnosis?
a) Worsening symptoms at night
b) marked interpersonal aggression
c) history of cruelty towards animal
d) history of drug use

93) Woman whose son is a drug addict, beats her every day. She is afraid and presents to
you. What will u do for the safety of the woman?
a) Tell her that you are obliged to inform the police
b) Go to the police
c) Tell her to go to refugee shelter
d) Advise her to go support group
e) Send the son to correctional facilities

94) old patient who drinks about 10-12 standard drinks per day and also smoker,
underwent some surgery. on 3rd post op day he developed agitation ,O2 saturation
88%.no fever. what next ?
a) blood alcohol
b) blood glucose
c) CTPA
d) urine culture
e) CXR

95) A woman came for interview asked about appetite she said one doesn't get cheese from
moon and I came by bus What is diagnosis
a) Behavioral problem
b) Depression
c) Delusion
d) Depersonalization
e) Dissociation

96) Diabetic patient well controlled with mild renal impairment (given creatinine level-
?120). Which of the following risk will have in baby?
a) Macrosomia
b) Renal agenesis
c) Intrauterine fetal demise
d) Low Birth weight
e) IUGR

AfraTafreeh.com
Get more from: MplusX Qbank (https://member.mplusx.com/)
Facebook Page: (https://www.facebook.com/mplusxqbank/)

97) 75 years old man present with severe retrosternal chest pain. pain is preceded by
vomiting. with BP 100/70 mmHg, dull on percussion and reduced breath sound at base
of the lung. he has HTN DM asking most diagnostic test?
a) non-contrast CT
b) electrocardiogram
c) chest X-ray
d) gastrograffin swallow

98) huchison freckles picture was given.asked next


a) local excision
b) excision with 2cm margin
c) immuiquimod cream
d) liquid nitrogen

99) A man with history of limb claudication on 100 meters relieved by rest, on
examinations there was absent left femoral pulse and absent dorsalis pedis pulse, ABI
was done and it was 0.25. What is the most appropriate test leading you to the
diagnosis?
a) Ct angiography
b) compression Doppler ultrasound
c) MR angiogram
d) X-ray
e) arteriography

100) An old women who is widow who seems well in past but since she became a widow
and she moves to the house and start to stay alone one of neighbor had noticed her
strange behavior. She starts to dig some ground in front of the house and when
neighbor reached near her and looked at her, she then aroused him and get aggressive
and accused him and all neighbors as imposters. After that she became calm down and
agreed to be seen at the medical clinic. Which one of the following in her MMSE will
help u to get a diagnosis?
a) Thought forms
b) Mood
c) Delusion
d) Memory
e) orientation

101) aboriginal woman come with some disease. can't remember exact scenerio.GP
offered her treatment but she wants to take their ritual herbs for that disease instead
of the medicine gp prescribed. what is the next step?
a) allow her
b) admit her under mental health tribunal
c) include aboriginal officer

102) pt presented with fever. 38.with moderate ascites, gynecomastia , spider nevi,
flapping tremors. vitals given. whats ur initial step of management of this patient ?
a) full blood examination

AfraTafreeh.com
Get more from: MplusX Qbank (https://member.mplusx.com/)
Facebook Page: (https://www.facebook.com/mplusxqbank/)

b) ammonia level
c) abdominal USG
d) LFTs
e) Abdominal paracentesis

103) Patient who has HTN with DM & AF on metformin,Statin and warfarin and
started amiodarone 3 days earlier suddenly got swelling at right thigh for 12hours
which is progressively increasing and become painful and got fever 37.4.Right thigh
size is 4 cm larger than left.Diagnosis?
a) rhabdomylosis
b) DVT
c) Cellulitis
d) drug interaction
e) hematoma

104) child with asthma. On exam urinary ketones found and glucose high. Next to
follow up
a) Culture and sensitivity
b) Glucose level
c) HbA1C

105) A 28 year old girl came for CA breast screening since her paternal aunt has CA
breast and ovarian cancer diagnosed at 48 years old. What is your appropriate
management?
a) BRAC1 muatation screening
b) Reassurance
c) Early mammogram screening
d) Self breast awareness
e) conduct genetic risk assessment

106) 45 year old woman who smokes a lot. She is on COCP. Her BP is 155/- mmHg. She
doesn’t want to change her method of contraception since it has controlled her heavy
bleeding during menstruation and she doesn’t want to conceive. What is the most
appropriate contraception for her?
i) Tell her to quit smoking so that she can continue to take OCP
ii) Tell her to use condom
iii) Offer her copper IUCD
iv) Change her onto POP

107) Old female with depression for 6 wk after dead of her partner ,sleeps badly, she
lives alone now , she had the same symptoms in the past when she lost her child , what
is the prompt Rx?
a) Olanzapine
b) ECT

AfraTafreeh.com
Get more from: MplusX Qbank (https://member.mplusx.com/)
Facebook Page: (https://www.facebook.com/mplusxqbank/)

c) midazolam
d) venlafaxine
e) temazepam

108) man aged 64 yr. his psa is 3.8ng now.last year it was 2 or 1.8.he did 12 biopsies this
time and one showed a focus of adenocarcinoma and gleason score 4 . What is the
management for this man?
a) radical prostatectomy
b) external beam radiotherapy
c) orchidectomy
d) continuous surveillance

109) 40 year old woman after lifting some heavy thing , develop pain in the right
buttock and tenderness in her back at L4-5 and difficulty and restrictions of
movement in extension and flexion and rotations , what will you give beside analgesic?
(Dx-muscle sprain)
a) spinal xray
b) spinal MRI
c) bed rest
d) referral to orthopedics
e) keep active

110) 80 year old man presented with loin pain, urine examination showed hematuria
+++. What is the most appropriate Mx?
a) XRay
b) IVP
c) Retrograde pyelography
d) Urine culture(next)
e) CT abdomen

111) year old retired farmer presented with left groin lymph node enlargement. The
biopsy specimen shows squamous cell carcinoma. what is the primary site?
a) anus
b) rectum
c) prostate
d) left leg

112) Young couple with infertility. Female with one children in previous marriage and
female investigations were normal, what in history you will ask the male patient that
will be most useful to guide u to cause of infertility?
a) How much alcohol do u drink?
b) Did u get chicken pox infection for once while u were young?
c) Do you use Marijuana?
d) Do u have regular unprotected sex?
e) Do you use ecstasy?

AfraTafreeh.com
Get more from: MplusX Qbank (https://member.mplusx.com/)
Facebook Page: (https://www.facebook.com/mplusxqbank/)

113) A patient with spiking fever, SOB and was inserted prosthetic heart valves 3
months ago. And 3 blood cultures done dx as due to staph aureus IE, and given
flucloxacillin for treatment. The fever subsides after 2 weeks of treatment, but SOB
still persists, diastolic murmur at the left sternal edge? What will u do for
investigation?
a) CXR
b) troponin
c) blood culture for every week
d) trans thoracic ECHO

114) A 35 year old girl is on a low budget trip to India. She has received one oral polio
vaccine, measles vaccine at 18 months of age. Tetanus vaccine at 25 years of age.
Apart from Hep A and influenza vaccine. What will you give her?
a) Polio and DPT
b) Polio and MMR
c) Polio, MMR and DPT

115) Women on HRT for 6 years, well controlled of her symptoms. DEXA scan shows-
femur- 1.7 vertebra- 1.2 . how will you manage her?
a) Continue same treatment
b) Add vitamin D and calcium
c) Cease HRT
d) Change HRT to alendronate
e) Add alendronate

116) Post menopausal lady with reflux esophagitis. DEXA scans of her femur shows -
2.5 and vertebra -2.5. What is the most appropriate management?
a) alendronate
b) vit D and Calcium
c) Residronate
d) strontium relanate

117) Post menopausal lady with reflux esophagitis and she’s on PPI. DEXA has been
done and it shows severe osteoporosis. What is the most appropriate management?
a) Alendronate
b) Vit D and Calcium
c) Residronate
d) IV zolendronic acid

118) Patient came back from asia and low grade fever and cough for – weeks. What is
the most likely diagnosis?
a) Lymphoma
b) TB
c) Viral pneumonitis

AfraTafreeh.com
Get more from: MplusX Qbank (https://member.mplusx.com/)
Facebook Page: (https://www.facebook.com/mplusxqbank/)

d) Pulmonary embolism

119) Post colectomy opt pt oliguria without catheter. most appropriate mx?
a) Abd xray
b) blood culture
c) S electrolyte and creatinine
d) Bladder scan

120) patient with COPD was admitted for 3 days in hospital. bronchodilator was given
8 hourly, ipratropium bromide, and oral prednisolone. sputum was of thick yellow
color. Despite the treatment, he still has sever respiratory distress but he is alert.
Arterial blood gas examination are as follow
pH - 7.35 (7.36-7.44)
PCO2 - 50 (30-45)
PO2 - 80 (80-100)
HCO3 - 35 (22-26)
What is the next appropriate step?(Dx-type 2 respiratory failure)
a) bronchodilator 4 hourly
b) change antibiotic
c) change to IV hydrocortisone
d) arrange refer to ICU for intubation
e) CPAP

121) patient with BMI- 35 what is the best management for long term weight
maintenance together with exercise?(jm894)
a) low CHO diet
b) 4000 kJ/day
c) Lipase inhibitor(orlistat)
d) avoid all alcohol

122) Restless leg syndrome scenario. What is the next investigation?


a) Sleep studies
b) Nerve conduction test
c) Iron studies

AfraTafreeh.com
Get more from: MplusX Qbank (https://member.mplusx.com/)
Facebook Page: (https://www.facebook.com/mplusxqbank/)

123) Patient with claudication pain. Doctor advises surgery, but patient not fit to do
surgery. Femoral bruits +. He has DM, hypertension and obese. What will you advice
for long term management to reduce his claudication symptom?
a) Reduce smoking
b) Supervised exercise
c) Hypertension control
d) DM control
e) Weight reduction

124) year old woman had her last menstruation 8 months back. She is complaining of
hot flushes and insomnia. What is the best treatment option for her?
a) Cyclical HRT
b) Continuous oestrogen
c) Estrogen patches
d) Continuous Progesterone for 30 day
e) cyclical oestrogen with 12 days progesterone

125) 52 years old lady with involuntary loss of urine whenever she laughs or sneezes
with mild rectocele, but there is no cystocele. Urodynamic study has been done and it
showed loss of urine with low intravesical pressure. She was taught pelvic floor
exercise and symptoms have improved with it. But she is starting to embarrass when
she goes out in public. What is the next step to treat her condition?(stress
incontinence)
a) Weighted vaginal cones
b) imipramine
c) surgery for rectocole
d) Bladder neck suspension surgery

126) A mother came with her two children 2 yr and 7 yr. Mother's brother has
haemochromatosis. What screening test advice would you offer?
a) Screen mother and kids
b) Ask her to come with her husband
c) Screen only the children
d) Screen only mother but not the kids

127) 12 yr old girl living at foster care because she was removed from her parents
because they r drug abuse since she was a toddler. She needs help in school because she
can't read and write. She has good collection of her toys and play with her imaginary
friend. Her foster parents doesn't like that she only eats some kind of food. What is
the immediate risk of this child?
a) Schizophreniform disorder (long term)
b) Anorexia nervosa
c) Substance abuse
d) Schziphrenia girl
e) Sexual abuse

AfraTafreeh.com
Get more from: MplusX Qbank (https://member.mplusx.com/)
Facebook Page: (https://www.facebook.com/mplusxqbank/)

128) Post renal transplant patient . Done 8 months ago. Now present with headache
and neck stiffness. X ray show rounded lesion in the right lung. What is the most
likely diagnosis?
a) Lymphoma
b) TB
c) Nocardiosis
d) Aspergillosis

129) Pt with active RA on methotrexate and chronic pyelonephritis ,feels tired and
lethargic. labs done which showed
Hb 9.5 g/dl
Mch low normal
Serum iron decreased
S.ferritin normal
Tibc decreased
What is treatment of choice
a) packed RBCs
b) iv iron
c) low dose corticosteroid
d) Folic acid
e) erythropoietin injection

130) Case of community acquired pneumonia was treated and got well on day three
after giving 2 antibiotics (IV augmentin, oral roxithromycin) and given enoxaprin. On
day five patient started having fever 39F, chills and rigors associated with crepitation
in right lower zone. Foul smelling purulent white nasal discharge.
a) Hospital acquired pneumonia
b) Drug hypersensitivity
c) Iv cannula bacteremia
d) Pulmonary embolism
e) empyema

131) The patient has 8 months history of joint pain in the wrist and the ankle and
minimal pain in other joints but now the patient comes with morning stiffness of both
wrists for 1 to 2 hours.
Her lab results as follow.
Hb reduced
MCV slightly reduced (nearly below lower margin level)
ESR 70 (sure for this level)
a) NSAIDs
b) Prednisolone
c) Hydroxychloroquine
d) Etarnarcept
e) methotrexate

AfraTafreeh.com
Get more from: MplusX Qbank (https://member.mplusx.com/)
Facebook Page: (https://www.facebook.com/mplusxqbank/)

132) A man presents with bradykinesia, rigidity, given risperidone for visual
hallucinations, he also complaints of fluctuating forgetfulness and signs of
parkinsonism. What is the diagnosis?
a) Parkinson’s disease
b) Alzheimer’s disease
c) Drug induced psychosis
d) Frontal lobe lesions
e) Lewy body dementia

133) What is the most important indicator in finding health risk of obesity?
a) BMI
b) Waist hip ratio
c) High LDL
d) Low HDL
e) waist circumference

134) 8 years old boy brought due to behavior changes. Noticed to have blank stare, with
fidgeting of right hand, head twitching to right side, sometimes chewing. These occurs
in clusters for 3-4 days in a week, then symptoms free & back to normal for several
weeks episodes lasts for 40 to 60 seconds and he remain confused for sometimes after
the episode. Diagnosis?
a) Absence seizure
b) Juvenile myoclonic epilepsy
c) Generalized tonic clonic epilepsy
d) Temporal lobe epilepsy
e) Tourette

135) post operative pt, oliguric with indwelling catheter, fever 38.5, what is ur most
appropriate management ?
a) abd x ray
b) serum electrolytes and Cr
c) abd us
d) blood culture

136) Woman with a history of hysterectomy and DVT. Now complaining of hot flushes
and insomnia. What will you give her?
a) Oral low dose estradiol
b) Progesterone only pills
c) HRT
d) Transdermal oestrogen and progesterone
e) Transdermal estradiol

137) 8 yrs old with type 1 diabetes. on small n intermediate acting insulin. every
morning high glucose levels. what to do ?(somogy phenomenon)
a) check early morning insulin levels
b) increase the evening intermediate insulin
c) give insulin before breakfast
d) give another dose before sleep

AfraTafreeh.com
Get more from: MplusX Qbank (https://member.mplusx.com/)
Facebook Page: (https://www.facebook.com/mplusxqbank/)

e) check blood glucose levels at 2:00am-3:00am for 3 days

138) 14 years old girl lives with family has now become sexually active comes to you for
contraception advice and tells you not to inform her parents. What will u do?
a) Give her prescription of contraception
b) Tell her she needs parents’ permission
c) Inform parents
d) Prescribe her OCP and Tell her to use condom to her partner
e) Inquire about the identity and age of the partner

139) A patient (I think middle aged) presents with neck swelling which was gradually
increasing in size. The patient is now complaining of change in voice and stridor.
Difficulty in breathing when he lies down. There is dullness to percussion at the base
of sternum. Most likely diagnosis?
a) Papillary carcinoma
b) metastatic carcinoma
c) follicular cyst of the thyroid
d) multinodular goiter with retrosternal extension

140) Scenario of Ulcerative colitis. On colonoscopy friable mucosa 5cm. What to give?
a) Sulfasalazine

141) Patient taking amisulpride, complaints of gynaecomastia and sexual dysfunction.


What to give?
a) aripirazole
b) olanzapine
c) clozapine
d) quetiapine

142) a scenario of mesenteric ischemia , what most useful to confirm ur diagnosis beside
CT ?
a) abd x ray
b) abd us
c) serum lipase
d) serum lactate

143) Patient who is a known diabetic and hypertensive currently on ramipril and
metformin for the past 2yrs. 2 weeks ago she developed an upper respiratory tract
infection and took amoxicillin for three days and some vitamin c tablets. Now
developed tongue swelling and bilateral wheezing on chest auscultation. Cause is the
most likely dug which can cause this presentation
a) amoxicillin
b) Metformin
c) Vitamin C tabs
d) ramipril

144) Insufficient milk production, reason?

AfraTafreeh.com
Get more from: MplusX Qbank (https://member.mplusx.com/)
Facebook Page: (https://www.facebook.com/mplusxqbank/)

a) insufficient fluid intake


b) not putting for long time during feeding
c) exhausted
d) poor positioning
e) not putting for sufficient frequency

145) Child presented with UTI, on USG the right kidney is smaller than the left. What
is the best investigation to assess renal function?
a) DTPA
b) Urine culture
c) CT scan
d) DMSA

146) Waldenstorm macroglobenemia patient came with rigors and fever and right lower
lobar pneumonia or some infection like this admitted and started ampicillin and after
minimal time generalised vesicular rash appeared asking test
a) Allergy test
b) skin biopsy

147) sudden red eye photophobia blurred vision on left eye reduced to conting fingers.
right eye ok. Increase pressure on tonometry approx 25
what is the long term mx?
a) trabeculectomy
b) topical pilocarpine!
peripheral iridotomy

ADVISE:
• do read HB and JM properly. Do recalls of last 6 months – 1yeaar. 6 months is
must, if can do , do upto last 1 year’s recalls
• Make up study group or atleast study with a study partner.
• For last few months of prep: divide your study time for HB and JM and recalls
and revise HB alleast 2-3 times and JM twice.
• Go through all associated pictures carefully

Perth recalls
1. Heart failure case man with AF on Ramipril, metoprolol, digoxin. Went on trip for 2 weeks
and stopped his medication. Now with bilateral oedema up to knee in legs. And ankle
swellings Lungs is clear. And the patient is not dyspneic,
What is the most appropriate management?
A- Frusemide
B- Recommence all his medications

AfraTafreeh.com
Get more from: MplusX Qbank (https://member.mplusx.com/)
Facebook Page: (https://www.facebook.com/mplusxqbank/)

C- Ramipril
D- Digoxin
2. Women comes to you in burqa for normal routine examination what additional test will
you perform..
a. Vit b12
b. Vit D
1. 3. There was a question about Forest Plot. Looks like this but the numbers at the base are
0.001, 0.01, 1, 10, 100. They conducted a study about
The graph measures the presence and absence of pain. What does the plot mean?
a. DRUG X is more effective than Drug Y
b. DRUG Y is more effective than Drug X
c. Forgot

1. 4 A man aged 64 yr. his psa is 3.8ng now.last year it was 2 or 1.8.he did 12 biopsies this
time and one showed a focus of adenocarcinoma and gleason score 4 . management
asked?

a. A) radical prostatectomy
B) external beam radiotherapy
C) orchidectomy
D) continued surveillance

2. 583 year old patient, living alone was asking help for her will. She always visit her GP
and buy her gifts. What should be her action?
A) Assess testament capacity
B) Help her with the will

AfraTafreeh.com
Get more from: MplusX Qbank (https://member.mplusx.com/)
Facebook Page: (https://www.facebook.com/mplusxqbank/)

C) tell her that her daughter can be beneficiaries


D) forgot

3. History of appendicetomy & cholecystectomy presented with 3 wk history of abd pain,


distension, BS exaggerated , CT abd image given
a. Sigmoid volvulous
b. Adhesive IO
c. CA Sigmoid
d. Pseudo obstruction

4. 3115.Old man 75 years of age has 3 week history of constipation. He presents with mild
abdominal tenderness and on rectal examination there is no faeces in the rectum. On CT
there is cut off at the sigmoid colon and dilation of the ascending, transverse and descending
colon. Which of the following is the most likely diagnosis? (no ct given)

a. Sigmoid volvulus

b. Colon cancer

c. Caecal volvulus

5. flial chest Q,CT given(hemopneumothorax),MVA,#rib,painfull shallow breathing ,O2 given by


face mask,whats next?

a.morphine

b.intubate

c.drain

6. Young girls 9 years of age with history of recurrent URTI presented after sore throat.
Urinalysis showed blood (+) , protienuria (+) . Now she came after one week. Urinalysis is
unchanged except for Blood (+) which is non-glomerular and there are no casts in the urine.
Which of the following is the next best investigation?

a. Urine culture

b. DMSA

c. Renal Ultrasound

d. Bladde Ultrasound

e.asot

AfraTafreeh.com
Get more from: MplusX Qbank (https://member.mplusx.com/)
Facebook Page: (https://www.facebook.com/mplusxqbank/)

7. Child presented with UTI, on Ultrasound right kidney smaller in size smaller than left. What is
the next investigation?

a. DMSA

b. Urine culture

c. CT scan

d. DTPA

8. ***long scenario(looks like MND)man comes with clumsiness of


hands,fassciculations,proximal muscles weekness,foot drop,ix?

a. EMG

b. MRI spine

. melanoma face recall----local excision

18. COPD OLD RECALL,pao2 decrease,whats next?

How much the relative risk increase in people not taking ASA to get coronary event?
1%
2%
100%
200%

Coronary event Present Absent

No Aspirin 2 98

Aspirin 1 99

2/(2+98) 2/100
Relative Risk (RR) = 1/(1+99) = 1/100 = 2

RR is more than 1 here,

that gives RR increase (RRI) = RR -1

= 2 -1

= 1 (100%)

RRI of 1 means 100% more likely to get coronary event in those who do not take Aspirin.

10. Husband beating up the wife and hostile. Asking for previous significant history to make the
diagnosis:

AfraTafreeh.com
Get more from: MplusX Qbank (https://member.mplusx.com/)
Facebook Page: (https://www.facebook.com/mplusxqbank/)

Child sexual abuse

Cruelty to animals during late childhood

. Q.Mother brings her 7 months old son with complaints of bleeding from mouth and gums.she told
you that he bumped his head somewhere so that he presented like this, on examination the child
has torn frenulum, old bruises on shins and forehead. No hepatosplenomegaly. No petechial rash.
What is the most likely diagnosis?

A-factor 8 deficiency

B-non accidental injury

C-Von willebrand disease

D-leukemia

.borderline PD scenario,young girl,angry,h/o unstable relationship,what to give?

Ans.dialectal therapy

92-old patient found unconscious at home brought by son to emergency department . the patient
was taking tablets methadone, oxycodone , others , patient has constricted pupil on examination
which of the followings is the likely cause?

A-methadone

B-oxycodone

C-may be cocaine don not remember other options

. Pt with RA she takes ibuprofen and methotrexate to control her disease ,they mention the time it
was years for both of drugs , pt complains of some symptoms , and her labs are given ALT ,AST GGT
all were high , the Q is asking about the cause of this ?

A-Methotrexate induced hepatitis

B-Ibuprofen induced hepatitis

C-Autoimmune hepatitis

DViral hepatitis

. Pt with hoarseness and right ant swelling in the neck move with swallawing with dullness
percussion on the upper part of sternum dx

Multinodular goiter with retrosternal extension

Papillary ca

Follicular cyst

AfraTafreeh.com
Get more from: MplusX Qbank (https://member.mplusx.com/)
Facebook Page: (https://www.facebook.com/mplusxqbank/)

. Woman with prescription of HRT.She has been taking HRT for 6 years now and no complaints.What
will you do?FEMUR 1.7 SPINE 1.5

Trial of ceasing HRT

REDUCE HRT

CONTINUE HRT

HRT TO ALENDRONATE

dysmenorrhoe & menorrhagia---

Ans.mefenamic acid

Patient with decreased Hb with cyclical menorrhagia. Hysteroscopy and D and C done. Both normal.
Wts next

a. levonorgestral releasing IUCD

b. progesterone from day 15-25 of cycle

c. oral tranexamic study

D.COCP

Post hysterectomy patient with history of DVT post operative after hysterectomy. Now comes with
hot flushes. Which HRT is best?

Low dose Oral estradiol

Oral estrogen plus Progesterone

Oral Progesterone alone

Transdermal estrogen patch

Low dose Oral estradiol

.man with sexual difficulty on resperidone ,what to do?

Ans.aripiprazole

.prostate ca scenario.DRE was irregular & enlarged prostate,next inv?

PSA

PET SCAN

TRUS

AfraTafreeh.com
Get more from: MplusX Qbank (https://member.mplusx.com/)
Facebook Page: (https://www.facebook.com/mplusxqbank/)

. Picture of a palm fascia thickening and fibrosis with skin puckering associated with the ring finger
and litter finger flexion (Dupuytren’s contracture) . Which of the following is the most common
cause?

a. Farm worker

b. Chronic infection

c. Vibrating tool damage

d. Family inheritance

e. alcoholism

Pregnant patient with increased BP 145/90 blurring of vision HEADache since 24 hours . upper right
quadrant pain.dx ??

A.preeclampia b.cholilitheasis

.SCC Q,--Left leg HBQ 3.392

37.gout with CRF scenario

Ans.prednisolone

A lady with BMI of 35 how will u manage her in addition to exercise,weight reduction and diet long
term mx ?

a.Involve in sports

b.Low carb food always

c.Lipase inhibitor

d.Diuretics

e.Regular review

. Child 5 yrs fell on garden bed has multiple superficial lacerations he has hx of 2 doses of vaccine at
2 nd 4 mntgs wht next

Dpt and booster after 2 mnths

Tetnus toxoid nd ig

Dpt and ig

****. Primigravida gave birth at term to 3200g baby, normal vaginal delivery without any
complication and there was slight meconium staining of liquor. Baby was normal at birth with

AfraTafreeh.com
Get more from: MplusX Qbank (https://member.mplusx.com/)
Facebook Page: (https://www.facebook.com/mplusxqbank/)

normal heart rate. After 1 minute suddenly stopped breathing, HR decreased, cyanosed. What is the
most appropriate next step?

a) Intubate

b) Nasopharyngeal aspiration of meconium

c) Bag and mask ventilation

If MAS occurs, your newborn will need immediate treatment to remove the meconium from the upper
airway. After delivery, your doctor will immediately suction the nose, mouth, and throat. Then, a tube will
be placed in your newborn’s windpipe (trachea) to suction the fluid containing meconium from the
windpipe. The suctioning will continue until no meconium is seen in the material removed.

If your newborn is not breathing or has a low heart rate, your doctor will use a bag and mask to help them
breathe. This will deliver oxygen to your baby and help inflate their lungs. Your doctor may need to place
a tube in your newborn’s windpipe to help them breathe if the infant is very ill or not breathing on their
own.

After emergency treatment has been provided, your newborn may be placed in a special care unit to
observe their breathing. Additional treatment may be needed to avoid complications of MAS. Five
common treatments include:

antibiotics, such as ampicillin and gentamicin to prevent or treat an infection


the use of a ventilator, a breathing machine, to help your infant breathe
extracorporeal membrane oxygenation (ECMO) if your baby is not responding to other treatments or has
high blood pressure in the lungs (for this treatment, a pump and machine that performs the function of
the lungs do the work of your newborn’s heart and lungs so that these organs can heal)
oxygen therapy to make sure there is enough in the blood
the use of a radiant warmer to help your baby maintain body temperature....Rihan Aswad

42. What’s the most important marker in finding health risk of obesity?
- BMI
- waist circumference
- waist hip ratio

45. Primigravida gave birth at term to 3200 g baby, normal vaginal delivery without any
complications and there was slight meconium staining of liquor. Baby was normal at birth with
normal heart rate. After 1 minute, suddenly stopped breathing.HR 40. Cyanosed. No response on
stimulation. What is the most appropriate next step?
a. Intubate
b. Nasopharyngeal aspiration of meconium
c. Bag and mask ventilation

48. Restless leg syndrome scenario. Investigation asked:

a. iron studies

AfraTafreeh.com
Get more from: MplusX Qbank (https://member.mplusx.com/)
Facebook Page: (https://www.facebook.com/mplusxqbank/)

b. sleep studies

c. nerve conduction test

scaphoid xray fracture of bone treatment ??

plaster cast

Boy in MVA , seatbelt injuty to left chest. On examination , dimished sound on left sided , condition
not improving . on arrival , chest and abdomen pain with BP 90/60 , HR – 110/min. Give IV fluid in
hospital. After one hour , he complaint of shoulder pain with BP 80/60 , HR . 120/min .no
pneumothorax seen in xray. Mx ?

a) Chest drain
b) Intubation and IPPV
c) Needle thoracocentesis
d) Laparotomy
e) Laproscopy

Woman with CCF , taking digoxin and perindopril regularly. Long list of lab result given. ECG show
pulmonary congestion(given) and she has progressive dyspnea

a) Add furusemide
b) Add
c) Cease digoxin and give K+ supplement
d) Cease perindopril and give K+ supplement
e) Urgent rhymthm inversion?

Amedex ans

Man whose hand deeply injured by screw driver , 5 weeks ago, he found supercial laceration and
give diphtheria tetanus toxoid , now , the hand is swollen and painful. What to do beside antibiotics?
a) Elevation
b) Aspiration
c) Tetanus toxoid
d) Tetanus immunoglobulin only
e) Dpt

57yr male with waldenstorm macroglobulinemia (pic given)and myelodysplastic syndrome


developed pneumonia. Penicillin given. After 2 days he developed maculopapular rash with some
pustular lesion .What would lead to definitive diagnosis:
A. Skin biopsy
B. Immunoflourescense of lesion
C. Blood culture
D. Marrow biopsy
E. swab of lesion

AfraTafreeh.com
Get more from: MplusX Qbank (https://member.mplusx.com/)
Facebook Page: (https://www.facebook.com/mplusxqbank/)

. A 44 years old man attends your OPD with complaint of increasing swelling over last 12 hours on
his right thigh. He is a diabetic and hypertensive and has been previously operated 6 weeks back for
a fracture femur. He is on Ramipril, Simvastatin, Warfarin, Amiodarone, NSAIDs, and antacids. On
examination you find that the right thigh is 4 cm greater in circumference than the left thigh and is
tender. His temperature is 37.8. Which of the following is the reason for his presentation?
A. Arterial embolus
B. Cellulitis
C. Deep vein thrombosis
D. Edema
E. Drug interaction

.lateral epicondilitis - initial management?

2 ctg qs stop synto 3.pleural effusion in child. 4.ct scan with apple core picture-sigmoid ca

.son abused mother, she is afraid what to do next - urge to go to police, provide shelter

17. Child with bleeding from frenulum,mother claims hit the table - nai

18.post op+oligouric+fever+Gen Abd pain. Initial Ix?

.chronic ra with pyelonephritis and low hb - erythropoiwtin injection

15.mitral stenosis x ray-bat wing appearance (pulmn hpt)

.patient on low budget travel what vaccination to give - polio,mmr,dpt

.litium toxicity symptoms

24.scenario on conversion disorder

25.patient on amisulride Dev gynacomastia. What alternative TX to give?

Mother living in mining town comes with her daughter for normal blood test. Her daughter’s test
show lead level 0.72 what will be your next appropriate step? (P 51)( K 1008)
A. Assess IQ testing of child (done in toxic level > 2.9 )
B. Educate mother about environmental risks
C. Refer for chelation therapy

AfraTafreeh.com
Get more from: MplusX Qbank (https://member.mplusx.com/)
Facebook Page: (https://www.facebook.com/mplusxqbank/)

D. Move the family to another town


E. Urine test to confirm dx

14 years old girl living with her parents comes for contraception mx ??

a.give ocp and condom advice for partnr

b.ask age of partner

inguinal HERNIA mx ?

diversification mx ?

diversification picture advice ??

NO ECG NO FUNDOSCOPY

MAN BEATS HER WIFE CHILDREN HAVE WITNESED IT MX ??

CALL CHILD PROTECTION SERVICE

Vit k queston

Patient on ACE inhibitor ,amoxicilline 2 days, C/O tongue pain ,protusion of tongue,wheezy
breathing from his chest,what is the cause?
a)Amoxicillin
b) ACE inhibitor

A patient with a painless neck swelling moves with swallowing. hoarseness and difficulty in breathing
at night…since 1 month. O/E swelling about 5cm in the anterior triangle of neck . dullness of
percussion at upper sternal border. most likely dx

MND goitre with retrosternal extension


anaplastic cancer
haemorrhage in nodule of MND
papillary carcinoma

Hep c positive mother how to avoid vertical transmission ?

Fetal scalp sampling

Ulfat Anwar
27/6/18

AfraTafreeh.com
Get more from: MplusX Qbank (https://member.mplusx.com/)
Facebook Page: (https://www.facebook.com/mplusxqbank/)

A 41 yr obese female, pregnant 32 weeks gestation, presents with upper abdominal pain since a few days,
bp-150/92. Occasional headache. All other examination- WNL. What is the diagnosis?
a) Acute cholecystitis
b) Pre eclampsia
c) Concealed placental abruption

2) Not sure about age, obese scavandian female for first antenatal check up(@ 8 weeks) . What will you do
?
a) OGTT now
b) OGTT @ 24-28 weeks
c) RBS @28weeks

3) Your female patient calls you, says husband is getting aggressive and violent. She is hiding in
bathroom while calling you and you can hear husband shouting in background, what will be your initial
appropriate advise?
a) Ask her to go to women’s shelter
b) Encourage her call the police immediately

4) You are a rural GP. You are known to both husband and wife and have good social relationships with
them. At a gathering, Wife secretly and tearfully confesses to you that her marriage is not working. What
will be your action?
a) Arrange appointment for a long interview
b) Arrange a couple interview
c) Send her to another GP
d) Console and embrace her now

5) A middle aged man comes in ED saying something about danger to his life foe which he has kept a knife
with him, although he know he shouldn’t. on asking, he evades about weapons. And refuses you to contact
his psychiatrist whom he saw last week. What will you do?
a) Contact psychiatrist against his will
b) Wait until he agrees to see his psychiatrist(not sure about exact words)
c) Observe
d) Other irrelevant options

6) Old age man case of metastatic pancreatic carcinoma. Complaints of pain, lethargy and constipation.
Lives alone, daughter makes a visit most of the evenings. He is sad and hopeless. What is most important
thing to access immediately in this patient?
a) His pain score assessment
b) His suicidal thoughts
c) Any drug abusive behavior
d) Others irrelevant

AfraTafreeh.com
Get more from: MplusX Qbank (https://member.mplusx.com/)
Facebook Page: (https://www.facebook.com/mplusxqbank/)

Because depression is present in pancreatic ca

7) You are a member of an infection control team of your hospital, you found out increased infection rate
for one surgeon with same procedures and surgeries. What will be your most appropriate action?
a) Talk to the surgeon
b) Talk to the head of surgery unit
c) Discuss with your infection control team
d) Inform the director

You remember that you prescribed pantoprazole to wrong patient yesterday. Now patient is stable, no
fresh complaint. What should be your most appropriate action?
a) Inform patient that you made a mistake
b) Tell patient that this is drug for acidity
c) Do nothing
d) Others I forgot

9) 9 months old child presented with abdominal distension, elder sibling had h/o RTI few days back.
Abdominal xray- paucity of air in right quadrant. What is the most appropriate treatment?
a) Air enema
b) Iv antibiotics
c) Others I forgot

Itersusseption due to itp

10) Elderly diabetic, what to check 3 monthly?


a) FBS
b) HBa1c
c) Fasting lipid profile
d) ACR

42

AfraTafreeh.com
Get more from: MplusX Qbank (https://member.mplusx.com/)
Facebook Page: (https://www.facebook.com/mplusxqbank/)

. 11) DKA scenario 14 years old male, what to check immediately in ED to make diagnosis? a) Urine ketones b)
USG abdomen c) ABG d) CT head

12) A young diabetic patient comes with h/o one episode of hypoglycemic attack after gymming for few
hours. Never had any episode before or after that. Asks advise for driving now?
a) Cant drive for 6 weeks
b) Cant drive for ?1year
c) Should avoid drive after vigorous exercise
d) Should drive only after fasting glucose levels maintained @3mmol/l

Normal range of glucose 35_5

13) One night you see your patient driving his car , when he is not allowed to drive at night. And you are
sure that it was your patient, not anyone else. What will be your most appropriate step?
a) Inform to driving license authority without knowledge of patient
b) Invite patient to discuss with him first
c) Ignore

14) MND scenario with mixed features of right upper limb and left lower limb. What is most appropriate
investigation?
a) EMG
b) MRI cervical spine
c) Nerve biopsy
d) ACE receptor antibodies

15) Middle aged man presents with pain from sole to great toe of right foot, which awakes him from his
sleep. O/E: inflammation from sole to great toe, non tender, active and passive movements are painless
and free. No sensory loss, SLR- positive @30 degree. Pedal pulses normal. No h/o dm/gout/arthritis.
Reflexes are normal. What investigation is most important for diagnosis?
a) Serum urate
b) MRI lumbar spine
c) Others I forg NH6Qot

16) Old diabetic patient , mentioned well controlled diabetes upto last week. Now presents with
(pic given- two infective ulcers @medial malleolus of left foot). Pulses normally palpable, no
sensory deficit. What will your next step after cultures and antibiotics?
a) Plain Xray foot
b) MRI foot
c) Arteriography
d) Duplex doppler of lower limb

HERE WE NEED TO RULE OUT OSTEOMYELITIS SO NEXT XRAY

AND BEST MRI

17) Ques with this foster plot telling about meta analysis for comparison between antibiotic X and Y. what
is your interpretation?
a) Antibiotic X is more effective than Y
b) Antibiotic Y is more effective than X
c) data is insufficient
d) Meta analysis not adequate to make any conclusion

18) PAP smear report: intermediate risk, HPV(not 16,18:mentioned) positive and LSIL present. What do to next?
a) PAP smear after 12 months b) Colposcopy within 3 months c) HPV after 12 months d) Cone biopsy within 12

AfraTafreeh.com
Get more from: MplusX Qbank (https://member.mplusx.com/)
Facebook Page: (https://www.facebook.com/mplusxqbank/)

months

19) Despite increased no of patients covered under cervical cancer screening (from68% to 85% now).
There is no decrease in cervical cancer incidence. What is the cause?
a) Screening test is not specific
b) Cervical cancer has very rapid course
c) Treatment not adequate

20) Father brings her girl child in ED , now father doesn’t want to leave her in ED , what to do?
a) Discharge and send her home with father
b) Ask if she needs her father to be with her in ED
c) Sorry forgot other options

AfraTafreeh.com
Get more from: MplusX Qbank (https://member.mplusx.com/)
Facebook Page: (https://www.facebook.com/mplusxqbank/)

Depands on age and severity of disease

21) Your colleague who is anxious and insomniac due to some board exam, asks for sleeping pill
prescription from you. What will be your most appropriate action?
a) Advise regular exercise and relaxation techniques
b) Prescribe him according to his wish
c) Ask him to consult his physician
d) Prescribe him melatonin agents as they have better efficacy

22) Patient TFT given: TSH on higher side, T3- low, what next investigation to do?
a) MRI brain
b) Thyroid antibodies
c) Thyroid scan
d) USG neck

23) Patient present with c/o shooting pain along the back of the leg and sole of foot. What additional
finding helps to localize the lesion?
a) Loss of ankle jerk
b) Loss of knee jerk
c) Loss of sensation on medial foot
d) Loss of sensation on medial leg

24) Woman comes to see a male physician. She need to get pelvic examination but she is reluctant to get it
done from male doctor, what should be your appropriate action?
a) Ask her if presence of female nurse would help
b) Ask her to come when female physician available
c) Send her to some another physician
d) Others irrelevant

25) Police brings in a drink and drive case, agitated, with blood alcohol level- 0.2%. what will be your
action to take appropriate history?
a) Observe until he gets stable to give history
b) Other options I forgot

26) Female forgetful and keep losing track of time, what to check?
a) Orientation
b) Insight
c) Constructional apraxia
d) Delusions

Wife brings Ptx (husband) with worsening Parksonism syx(marked bradykinesia,rigidity,lack of


self empathy and agitation) presenting with auditory and visual hallucinations comes to you for
advice.Ptx is currently on carba and levodopa 75mg with little to no improvement for more than a
year now. Whats the appropriate next step

AfraTafreeh.com
Get more from: MplusX Qbank (https://member.mplusx.com/)
Facebook Page: (https://www.facebook.com/mplusxqbank/)

A. Donezpil
B. Quietapine
C. Haloperidol
D. Increase Levodopa
E. Pramipexole

India

Hello everyone. I got around 60% recalls and the new questions
were also managable..Iam not able to clearly recall the new
questions. But most of them are same topic related. Most of my
recalls from Aug – Nov 2017 and also from my same month
exam questions June ’18 . Sorry for the late upload

1.melanoma eye pic refer to plastic surgeon

2.

baby after 6 hours of delivery develops jaundice. Cause

A) Neonatal sepsis

B) Hemolytic anemia

C) Beast feeding

1. Girl 16 years history of weight loss of 10 kgs, BMI 15. Her school performance
is good and recently fall in grade. What in history will help in diagnosis?
A. Episodic fatigue
B. Sleep disturbances
C. Feeling of worthlessness

AfraTafreeh.com
Get more from: MplusX Qbank (https://member.mplusx.com/)
Facebook Page: (https://www.facebook.com/mplusxqbank/)

D. Feeling of inferiority
E. Idea of guilt

1. A 30 years old woman with increasing dyspnea, dry cough since 2 yrs with
painful lumpy red swelling/skin lesions on legs and shins. On examination, there
are red, painful, lumpy lesions in leg and bilateral ankle swelling. ACE level ( 120
) increased. Which inv for confirming diagnosis? (No given X-ray)
A. CT scan chest
B. Skin biopsy
C. Blood culture
D. Aspiration
A woman works at a part-time job ( office cleaner ), and complaints of early
morning headache, frontal & bilateral, dull in character, varying in intensity, she
takes paracetamol & ibuprofen, which only cause relief for 2-3 hours. What is
the cause?
A. Drug rebound headache
B. Migraine
C. Tension headache
D. Cerebral tumor
E. Pre-menstrual headache
Scenario was clear drug rebound headache

2. patient taking amisulpride, complaints of gynaecomastia and sexual


dysfunction. What to give?
A. Aripiprazole
B. Olanzapine
C. Clozapine
D. Quetiapine

3. patient with history of claudication, smoke 30 pack cigarette per day, drink
alcohol , obese with diabetic history . he is not fit for surgery. He asked you for
the appropriate advice that will improve his symptoms of claudication?
A. Reduce smoking
B. reduce alcohol drinking
C. supervised exercise
D. control his hypercholesterolemia
4.
Most Common cause of low milk production
A. Low fluid intake
B. Less adequate baby feeding
C. Baby is not put on breast feeding frequently

AfraTafreeh.com
Get more from: MplusX Qbank (https://member.mplusx.com/)
Facebook Page: (https://www.facebook.com/mplusxqbank/)

A lady presented with her baby 5 times in 2 weeks at 10 weeks following


delivery - all normal, established breastfeeding, baby growing well on 90th
percentile. What relevant info will you ask the mother?
A. Past obstetric history
B. premorbid personality
C. history of psychosis
D. Thoughts of harming her baby

5. Found glucose and ketones in urine. Rbs was 9. Follow up with- hba1c
A child 6 YEARS old with history of asthma presented with upper respiratory
tract infection, urine examination showed very high serum glucose and
ketonuria of 2+. what is the most appropriate test to follow up this child?
A. HbA1C
B. serum creatinine and electrolyte
C. FBS
D. OGTT .
E. Blood gases

6. . Male with absent vas def. - check cf in both male female


Couple come for infertility problem for the last 12 months. On testing
examination and tests of the female are unremarkable. Male has azospermia.
And bilateral absence of vas deferens. Which of the following is most
appropriate before the starting the treatment of infertility?
A. No testing required as they can not have a child
B. Testing of both male and female for cystic fibrosis
C. Refer for IVF
D. serum fsh and lh for male

7. An old woman taken many tablets, now unconscious, pinpoint pupil, his son
brought her to hospital, and also tablets she might have taken, methadone,
oxycodone, buspirone patch
A. methadone B. oxycodone C. buspirone

24 months old baby brought by his parents for development assessment. What
should he have achieved?
A. Can pronounce 2 personal pronouns
B. Know family name
C. Know 4 colours

AfraTafreeh.com
Get more from: MplusX Qbank (https://member.mplusx.com/)
Facebook Page: (https://www.facebook.com/mplusxqbank/)

34 year old male with a history of fall on an outstretched hand with pain and
swelling. XRay was given, (showed scaphoid fracture. Very clear fracture line
through the middle)
What is the best treatment option for this patient?
a. Crepe bandage
b. Plaster cast
c. Analgesics
d. Compression screw
e. Plate fixation

8yr old brought due to behavior changes. Noticed to have blank stare, with
fidgeting of right hand, head twitching to right side, sometimes chewing. These
occurs in clusters for 3-4 days in a week, for 3O to 60s, Post ictal confusion, then
symptoms free & back to normal for several weeks. Asking Diagnosis?
A. absence seizure
B. Juvenile myoclonic epilepsy
C. Temporal lobe epilepsy
D. Tourette syndrome
8. A child with left kidney smaller than the right, what is the best measurement for
renal function?
A. DMSA - dimercaptosuccinic acid to see anomaly scar tissue due to vesico
reflux
B. DTPA
C. IV urogram
D. USG

A 74 year old man is being screened for prostate Ca. His PSA 2 years ago was
1.5ng/ml; now PSA is 3.8ng/ml. Only %5 of biopsy done showed
adenocarcinoma, Gleason score is 4. How will you manage this patient?
A. TURP
B. Radical prostatectomy
C. External beam radiation therapy
D. Androgen therapy
E. Active surveillance

9. Afib. Abdominal pain. Wat other test with ct- serum lactate
A scenario of an old man with history of atrial fibrillation and now comes with
severe abdominal pain. What other than CT angiography will you do?
A. CT abdomen
B. Serum lipase

AfraTafreeh.com
Get more from: MplusX Qbank (https://member.mplusx.com/)
Facebook Page: (https://www.facebook.com/mplusxqbank/)

C. Serum lactate

10. Patient with RA she takes ibuprofen and methotrexate to control her disease,
they mention the time it was years for both of drugs, patient complains of upper
abdominal pain, and her labs are given ALT, AST, GGT, even bilirubin all were
high, which of the following cause this condition?
A. Methotrexate induced hepatitis
B. Ibuprofen induced hepatitis
C. Autoimmune hepatitis
D. Viral hepatitis
E. Alcoholic hepatitis

11. Woman 25 years old is going to travel to India presented asking advice about
vaccination. She has previous 1 dose of polio vaccine, had DPT at birth, 2 and 4
months age, 1 dose of MMR at 13 month , and tetanus booster at 15 year age.
What vaccines should she received before travel?
a. MMR, diphtheria and tetanus
b. polio, diphtheria and tetanus
c. polio, MMR, diphtheria and tetanus
d. polio, MMR
e. MMR

12. ‘The moon is made of cheese, I came by bus’ Q .


On interview. Asking about the appetite. Patient told “That is concern with food.
Food which does not comes from moon, moon made of cheese. But I came her
by bus”. Which of the following will have in this patient?
A. Depression
B. Dissociation
C. Disorganized behavior
D. Delusion
E. Depersonalization

13. 52 yr old woman whose son is a drug addict, beats her every day. She is afraid &
presents to you. What will u do for the safety of the woman?
A. tell her that she is obliged to inform the police
B. informs the police
C. urge her to go to the refugee center
D. send the son to correctional facility
E. advise female support group

AfraTafreeh.com
Get more from: MplusX Qbank (https://member.mplusx.com/)
Facebook Page: (https://www.facebook.com/mplusxqbank/)

14. An 18 year old girl presents with , malaise and tiredness for months. No other
symptoms. Hb is 8.9, serum ferritin is 35, INR is 1.5. Calcium is 1.9. Long
question. What to do next ?
A. Blood transfusion
B. IV calcium
C. Injection vitamin k
D. Fresh frozen plasma
E. Vitamin D tablets
Here I was stuck.neither the hb nor she was bleeding..i was between iv
calcium and vitamin k…since she was at risk of bleeding I choose vita k.

Syphilis rpr qs... pt had rpr 0 at beginning then 1:200 then aft 6 months and now 1:2
Also mentioned were some tpha and some antibodies which were both positive at 6
months and at present...qs was asking wat does this mean

Pt has had syphilis 6 months back (amedex)

Pt has syphilis now

Pt had syphilis one year back

A good amount of stats questions which were ok to solve.Plz do the stats file which ll help
you.

I got LBA, infections like pneumonia ,abscess,cellullitis ect,psychiatry MDD,change of anti


depressants ect..absolutely no CVS questions. There were CNS questions almost all repeated
topics, child health with few developemental milestone questions…plz revise them, 2 and 4
mins synto questions…

Plz read the recall topic well…options are in twisted way

AfraTafreeh.com

You might also like